Download as pdf or txt
Download as pdf or txt
You are on page 1of 74

CHAPTER 3

Linear Programming-ll
Simplex Method

3-1. INTRODUCTION
While solving LP problem by graphic method, the solution space generally
consists ot aconvex ploygon bounded by constraint lines and the optimum solution
is searched only trom among the corner points of the solution space. However
when the number of variables and/or the number of constraints increases, it
becomes difficult to visualize the convex polyhedron representing the solution space,
in such
called Simplex and the graphical method cannot be successfully employed general
cases. Thus, an etficient computational procedure, is needed to solve the
class of linear programming problems. The Simplex method developed by George B.
linear
Datzing in 1947 is an iterative and an efficient method for solving
programming problems.
The Simplex method is an algebraic procedure that starts at a feasible extreme
point of the simplex (or convex set), normally the origin, and systematically moves
from one feasible extreme point to another until an optimum (or optimal) extreme
extreme (corner)
point is located. At each iteration, the procedure tests the one
another extreme point, of the
point for optimality, and if not optimum, choosesnon-negativity co:nditions of the
convex set that is formed by the constraints and
linear programming problem. Since the number of extreme points (i.e., corners or
leads to the
vertices) of the convex set of all feasible solutions is finite, the method
number of steps
optimum extreme point (i.e., optimum or optimal solution) in a finite proceed on to
or indicates that there exists an unbounded solution. Before we
of linear
explain the algorithm, it is essential to understand some characteristics
programming solutions.
is stated below
Alinear programming problem in the standard form required
Let x1, x2, ..., x, be the decision variables for a general LP problem. It is
to determine the values of these variables so as to :
Maximize (or minimize) Z = CjX2 + CzXy t .. + CXn
subject to the constraints
a11X1 + a12 X2 + ... + ain Xn bi

am) X1 + am2 X2 + .+ amn Xn Dm


and X2, X2, ..., x,20
where c, i= 1, 2, ... .n ) in the objective tunction are called the profit or cost
Coefficients, the constraints b, (i = 1, 2, .., m) are called resources and the
Constants a,,(i= 1, 2, ..., m ; j= 1,2, ..., n )are called structural coefficients.
It may be noted that the above form is nothing but asystem of m linear
equations in nvariables where m<n.A solution to this system is a particular set of
assumed bu the n
3.2
or positive)
which are variables
all the constraints are true.
negative i.e.,
(zero, equations is satisfied,
nvalues of m variablesin it assume non-negative

solutionyaluewhis elsh
system the m feasible solution is a
the
is feasible if all hand, a basic
Asolution
feasible and has at most
zero).
m positive valued variables (i.e., to say that the rermalnin.
Onthe other
positiveor variables are positive valued, the
less than m
If
m variables are zero).
basic feasible solution.
n - In practice the problem is not inthe standard form as stated above and has Io,
degenerate
said to be constraints are generally

than' or
either lessconstraint
standard form. The
converted into the 'greater than' signs
equality,
or in mixed form. In order to
we introduce
inequalconverities,t
slack or surplus variables,
w
ha
inequality
into
variables represent unused capacity and
variables represent slack
excess amount and the contribution (cost or profit co-effiien
terminology
In economic
is zero.
associated with them equal to" type is transformed into an
"less than or The slack variables can be equa
An inequality of the slack variable. production
capacittyhou
a non-negative 1 unit of
by the addition ofproducts, each requiring for its from the other, and fron
of as imaginary
a profit of zero, resources zero units of capacity each
of variable corresponds to the amount of unused capacih,
i.e., a slack yieldin
only one of the
which it is added.
for the constraint to as
a particular problem, the constraints m appears
Let in Sb,;i=1,2, ... , m.
ai, XË +a2 Xy t... +ain Xn
a suitable positive quantity S, to the left hand side, the inequalty
By adding
constraint can be written as :
bË :i=1,2, .. , m.
a,j X1 + a2 X2+ .... + ain Xn tS, =
fixed quantity. Moreover. slack
This new quantity s, is called slack variable, not a
variable may be interpreted as
Slack = R.H.S. - L.H.S:

An inequality of the "greater than or equal to" type is


transformed into an
equality by the subtraction of a non negative surplus variables. These surpls
variables represent the excess amount by which a particular requirement is met. I
the m constraints appears as
a1 X1 +a2 X2 t ... + ain Xn 2 b,;i=1,2, .., m
then by subtracting positive quantity s, from the left hand side, the inequaly
constraint can, therefore, be written as :
a1 X1+ a2 X2t.. + ain Xn - S, = b, ;i= 1,2, .., n
this new quantity is called the surplus variable or negative slack variable.
3-2. THE SIMPLEX ALGORITHM (Maximization case)
The solution steps of the simplex method can be outlined as follows :
Steps 1. Formulate the linear programming model of the real world problem,ie.
obtain a mathematical representation of the problem's objective function
Constraints.
2. Express the mathematical model of L.P. problem in the standard formby
adding slack variables in the left-hand side of the constraints and assign a zero
coefficient to these in the objective function.
LINEARPROGRAMMINGI
3-3
Thus we can restate the problem in terms of equations :
Maximize Z= C1X1+ c2 X2 +...
+CnX, Os1 + Os) +... +0sm
+
subject to the constraints
aj1X1+ a12X2 t... + a1n Xn +
S1
a21 X1 + a22 X2 t...t a2n Xn + = b1
S2 = b
am1X1t am2 X2 t... + amn Xnt Sm=bm
where X1, X2, ** Xn and s1, S2, Sm are non-negative. Note that the slack variables
have been assigned zero coetficients in the
objective function. The reason is that
hse variables typicaly contribute nothing to the
2 Design the initial feasible value of the objective function.
by setting :
solution. An initial basic feasible solution is obtained
X| = X) =... =X 0. Thus, we get s1 =
b1, S2 =b2,..Sm= bm
4 Set up the initial simplex tableau. For
computational
bo initial basic teasible solution, the constraints efficiency and simplicity,
of the standard LPP as well
obiective function can be displayed in a tabular form, called the simplex as the
shown below : tableau as
INITIAL SIMPLEX TABLEAU 1
C (contribution per unit) C,

Basic Value of Coefficient Matrix Identity Matrix Minimum


variables basic ratio
B variables
Xg/a,
bl=X) : S Sm
S
S) 1

1
B Sm e ...

0
Net contribution
G-Z :
per unit

The interpretation of the data in Tableau 1 is given as under. Other simplex


tableau will have similar interpretations.
() In the first row labelled "C,".we write the coefficients of the variables in the objective
tunction. These values will remain the same in subsequent tableaus.
(i) The second row shows the major column headings.
(ii) In the first column labelled "CR","we lists the coefficients of the current basic variables
In the objective function. Thus the coefficients of s1, $2, ..., Sm which are included in the first
programme,are written in the CB column.
(iv) In the second column labelled "Basic variables" (also called Product mix column) are
listed the basic variables.
values of the
(0) In the next column (also called Quantity column), we write the solution
basic variables.
(oi) the body matrix (under non-basic ariables) in the initial simplex tableau consists of the
Coefticients of the decision varjables in the constraint set. The entries in the body matrix can
0e interpreted as phusical rates of substitution.
OPERATIONS RESEARCH

3.4 (vi) Theidentity matrix(or basis matrix)(under S1, $2,


PROBLEMS
Sm in the initial simplex tablea
&

the
variablesin constraints set. Each
slack
represents the coefficients ofthebasic variables.
(viii) To find an
containsan identity
entryunder
matrix in the 2, row under a column, we multiply the entries o tha
Cp-column and add the products, i.e.
simplex taea
The Z, row entries wàll all
coresponding be equal to
entries of
zero inthe initialsimplextableau The Z, entry under the
column bythe current value of the objective function.
the
represent the decrease
column"in give
the value of objective function that would result one o he The
"Solution or quantity
entries
the solution were brought into the
solution. otherZ,
variables not included in the index row orThenet evaluation
calculation row,
"C-2,", called is
row labelled
(ix) The last or not the current solution is
optimum. of used to
simply involves
determine subtracting each 2, value from the corresponding Cj value for that colum.
whether
which is written at thetop of that column. We observe that C; Z, values are meaningtul for
because tor a basic variable, Z; =1xC.og o
the non basic variables only. This is
C-Z,=C -C=0.
Remark , The entries in the C -Z; row represent the net contribution to the objective unction
variables. A plus value
the respective column
that results
a greater
by introducing
contribution can
one unit each of
be made by bringing
the variable
would
for that
decrease
colurn
if onè
into
unit
the
of the tion.indicatnegates ive
soluvariable
A
tha
for the
by which contribution
value indicates the amount solution.
column were brought into the accounting prices).
row elements are also known as the shadow prices (or
Index
all
We test if the current solution is optimum or not. If the or elements
5. in the C;-Z; row i.e., index row) are negative or zero, then the current
entries
positive number, the current solution "
solution is optimum. f there exists some basis and replacing th
be further improved by removing one basic variable trom the
Some non-basic one.
improve the current feastble
6. Further, iterate towards an optimum solution. Todeparting variable) by a neu
the
solution, we replace one current basic variable (called
non-basic variable (called the entering variable). solution mix next. One way of doig
() We now determine the variable to enter into the C-Z,rowof the simplex
this is by identifying the column with the largest positive value in the
the key or pivot
table. The column with the largest positive entry in the C-Z, row is called entering
column (indicated by 1). The non-basic variable at the top of the key column is the
variable that will replace a basic variable.
(i) Next we determine the departing variable to be replaced in the basis solution. Ihis 5
accomplished by dividing each number in the quantity (XR) column by the corresponding
number in the key column selected in step 6) . We compute the ratio bj/aj b, /ag)
bm/amj The row corresponding to the minimum of these ratios is the key row or pivot rou
(indicated by ). The coresponding variable in the key row known as the departing on
will leave the basis.
intersection
(i) We identify the key or pivot element. This is the number that lies at the
of the key column and key row of the given simplex table.
7. Evaluate the new solution by tableau. After
constructing second simplex the new
identifying the entering and departinga variable, all that remains is to find
basic feasible solution by constructing new simplex tableau from the current one.
Now we evaluate or update the new solution in the following way :
thekey
(a) New values for the key row are computed by simply elementt of
row by key element. dividing every
can
(b) The new values of the elements in the remaining rows for simplextable
the new elements except
be obtained by perforning elementary row operations on all rows so that all
the key element in the key column are zero.
LNEARPROGRAMMING-I| 3-5

other words, for each row other than the key row. we use the formula :
Newrouw numbers -(Numberiin old rows)

-repondiag number above1


or below key element
Corresponding number in the 1|
row replaced in (a)

(c) New entries in the CB column and X column are entered in the new table of the
currentsolution.
(d Compute Z, and C-Z, rows. If all the numbers in Cj-Z row are either negative or
zero, an
optimum solution has been obtained.
8. If any of the numbers in C;-Z,row is positive, repeat the steps (6-7) again
until an optimum solution has been obtained.
Remark. Rules for Ties. In choosing the key column and key row, whenever there is a tie
between two numbers, the following rules may be followed :
(0 The column farthest to the left may be selected if there is a tie between two
row.
numbers in the index
(in The nearest ratio to the top may be selected whenever there is a tie between two
replacemernt ratios in the ratio column.
43. SIMPLEX ALGORITHM (Minimization case)
The various steps involved in using simplex method for minimization problems
are:
Steps 1. Formulate the linear programming model, and express the mathematical
model of L.P. problem in the standard form by introducing surplus and artificial
variables in the left hand side of the constraints. Assign a 0 (zero) and M as
coetficient for surplus and artificial variables respectively in the objective function. M
is considered a very large number so as to finally drive out the artificial variables out
of basic solution.
2. Next, an initial solution is set up. Just to initiate the solution procedure, the
initial basic feasible solution is obtained by assigning zero value to decision variables.
This solution is now summarised in the initial simplex table. Complete the initial
simplex table by adding two final rows Z, and C -Z. These two rows help us to
know whether the current solution is optimum or not.
3. Now we test for optimumityof the solution. If allthe entries of C -2, row
are positive, then the solution is optimum. However, this situation may come after a
number of iterations. But if at least one C - Z,s 0,the current solution can be
turther improved by removingone basic variable from the basis and replacing it by
Some non-basic one.
4. (0) Determine the variable to enter the basic solution. To do this, we identify
Ihe column with the largest negative value in the C;-2, row of the table.
(i) Next we determine the departing variable from the basic solution. If an
clal variable goes out of solution, then we discard it totally and even this variable
may not form part of further iterations. Same procedure, as in maximization case, is
employed to determine the departing variable.
O. We update the new solution now. We evaluate the entries for next simplex
iDle in exactly the sanme manner as was discussed earlier in the maximization case.
6.Step (3--5) are repeated until an optimum solution is obtained.
OPERATIONS REESEARCH:

3.6

The Big
sides
1. Il any
Introducing
M Method-
slack, surphus
constrainthave
obtain aconstraint
by -1 to problem

will reverse
and artificial
on
negative constants the
with a non-negative constant. (|f he conSItaint i
the inequality.)
the direction of
constraints.
pAOBLEMS&SO,
variables
right side,
equality, this In each S
slack variable
2. Introduce a artificial variable in each 2
3. Introduce a
surplus variable and an
ineach '='constraint.
constraint.
variable
artificialvariable A,, add -MA, to the objective function in case
For eachan
4.5. Introduce artificial
maximization and +MA, in case of minimization. Use the same constant M
(or all artli

variables.
tableau for the modified problem.
6. Form the simplex method.
Sove the modified problem by the simplex
7. problem.
Relate the solution of the moditied problem to the original
8. 0 If the modified problem has nosolution,thenthe original problem has no solution
0 If any artificial variables are non-zero in the solution to the modified problem, te
no solution.
the original problem has
Remark. The artiflcial variables are introduced for the limited purpose of obtaining an initial soluila
and are required for the constraints of 2type or the constraints with '=' sign. It is not relevant whete
the objective function is of the minimization or the maximization type. Obuiouslu. since artificialI variable,
must be driven out of
represent any quantity relating to the decision problem, they of infa t Jsten
do not if at all they do, it represents a situation assinnodh
and must not remain in the final solution (andhigh cost to them. Generally, a value M is
can be ensured byassigning an extremelynumber higher than any finite number. For this ro
artificial variable, where Mrepresents a are involved is termed as the 'Bio MMh
method of soluing the problems where artificial variables in the objective function a coefficient ofsM
When the problem is of the minimization nature, we assign for the problems with the objective functon i
each of the artificial variables. On the other hand, has a coefficient -M.
maximization type, each of the artificial variables introduced
3"4. Some Special Cases. Several special situations which one many encounte
during the application of simplex method are summarized below :
1. Non-feasible Solution. A linear programming problem may be unsoled
mathematically due to the contradictory nature of the constraints. Such an instante
is referred to as a non-feasible solution. A solution also non-feasible if an artifcid
variable appears in the basis of the solution purported to be optimum.
2. Unbounded Solution. It occurs in maximization problems, for example, wen!
solution variable can be made infinitely large without violating a constraint. I
discovered in simplex method if any iteration stage, all the entries in minimum tau
column are either infinite or negative.
5. Multiple Optimum Solution. This situation Occurs when there can beinfinite in
number of solutions possible for a given problem. This situation can be recognised
have a
a simplex method when one of the non-basic variables in the C-Zj row will
value of zero.
a zero
4. Degeneracy. Degeneracy is revealed when a basic variable acquires eitherthe
value (rather than a negative or positive value). In the final solution, numberof
number of basic variables is not equal to the number of or the
variables does not equal the number of decision constraints, The
instanceo

degeneracy is usually preceded by a tie for an existingvariables.


arbitrary
variable and anelement
selection for it. lf this is resolved bu a proper selection of the key
degeneracy can be avoided.
LINEARPAROGRAMMING 3-7

Problem 3-1. Amanufacturer has two products P and P2, both of which are
producedintwo steps by machines Mand M,. The process time per hundred for
products on the machines are :
the
M1 M2 Contribution (per 100units)
P1 4 5 10
P2 5 2 5
Available hours 100 80

The manufacturer is in amarket upswing and can sell as much as he can produce
f both products. Formulate the problem as LP model and determine optimum
Droduct mix, using simplex method.
Solution.
Steps 1. Formulation of the LP model :
Maximize (total contribution) Z = 10x1+ 5x,
subject to the constraints
4xj + 5x S 100 (constraint on machine I )
5x1 + 2x S 80 (constraint on machine II )
X1 20, X)20
where X]= The number of units of product P,
X2= The number of units of product P%
2. Convert the constraints to equations by introducing the slack variables. The
first step in the simplex method is to convert each inequality constraint into an
equation by adding a slack variable to each constraint. This can be done as follows :
Maximize Z = 10x1+ 5x2+0.s1+ 0.s2
subject to the constraints
4x1 + 5x2+ S1 = 100
5x1 + 2xg + S2= 80
X1,X2,S1, S2 20
Here, the slack variables, sj and s2 represent the idle or unused capacity (if any)
on the machines M, and M¡. Since this idle capacity will be either positive or zero,
the variables s1 and s are shown as non-negative. Further, the contribution per
hour of idle capacityon the two machines is taken as zero in the objective function.
3. Designing the initial feasible solution. The simplex method begins with an
initial feasible solution in which the basic variables xË and x are set equal to zero. In
other words, we are making an initial decision not to produce any one of the
products P and P,. This will also mean that the total given capacity on each
machine becomes idle capacity, i.e., S1 = 100 and s2 = 80.
4. Develop initial simplex tableau. The initial decision can easily be summarized in
atabular form known as Simplex Tableau or Simplex Matrix. An explanation of its
parts and how it is derived is given below
(a) Starting from left-hand side of the table, the column C, indicates the contribution per
unit tor the slack variables S1 and s ,Since the profits are not made on unused time, the
Contribution per unit for sj and s2 is shown as zero.
(b) The second column contains the variables (referred to as basic variables) constituting
the product mix.
(c) As the initial solution is not to produce any product, the total given capacity has
Oecome unused capacity, which is shown in the third column. Every number in the quantity
column must be zero or positive.
OPERA

3-8 INITIAL SIMPLEX TABLEAU


PARTS OF
TABLE1. Slack vartables
Realvarlables columns
column
Product
Profit mix
Quantity
per unit column
column
column 0 0

Basic
Solution
10
5

S
-Proilt per un
variables
B
values

5
1
Variable row
Rows il ustratina
tconst
Ëons rai(coelnt liequa
4
100
S
5 2 only) cient
80
Identity matrix con
Body matrix consis- sisting of coeffi
ting of coefficients cients of slack
of real product variables
variables
(d) The numbers inthe body matrix represent the physical ratio of substitution. For tis
to produce a unit of x1, we will have to give
table, the column means,
thatif we wish away 4
5 interpreted. In the
Similarly, the column can be same mànner
units of s1 and 5 units of sT. ratios of exchange
matrix represernt the physical
we can say that entries in the unit of body and unit matrices represent &
(e) The numbers at the top of the columns objective function.
coeffcients of the respective variables in the tablo
(9 Z, and C;-Z, (Index or net evaluation row). We may complete the initial simplexecononi
provide us with important
by adding two final rows. These last two rows whether the current solution i
information, including the total profit and the answer as to
optimum.
We compute the Z, value for each column of the initial solution in Table 2h
multiplying the zero contribution value of each number in the C; column by each
number in that row and jth column and then add up the products so obtained. The Z
value for the quantity column provides the total contribution (gross profit in this
case) of the given solution.
2, (For gross profit) = (Profit per unit of sj) x (No. of units of s)
+ (Profit per unit of s) x (No. of units of s)
= Rs. 0 x 100units + Rs. Ox 80 units = Rs. Oprofit.
2, (For column x) =Rs. 0(4) +Rs. O(5) = Rs. 0
Z, (For column x) = Rs. 0(5) + Rs. 0(2) = Rs. 0
Z (For column s1) = Rs. 0(1) + Rs. 0(0) Rs. 0
Z, (For column s2) = Rs. O(0) + Rs. 0(1) = Rs. 0
The C, - Z, number in each column represents the net profit, i.e., the proli
gained minus the profit given up, that will result from introducing 1 unit of each
product or variable into the solution. It is not calculated for the quantity column.
compute these numbers, simply subtract the Z, total for each column from the C
value at the very top of that variable's column. The calculations for the net profit per
unit (i.e., the C - Z rowor
Index row) in this case are:
X
Column
S S
C, for column Rs. 10 Rs, 5 Rs. 0
|2, for column Rs, 0
Rs. 0 Rs. 0
Rs. 0 Rs. 0
G -Z for column Rs. 10 Rs. 0
Rs. 5 Rs. 0
LINEAR PROGRAMMING|
3-9
TABLE 2.INITIAL SIMPLEX TABLEAU
Basic
variables Solution
values
10 5
COMPLETED
S 100 S
4 S
S) 80 5
5
2
Z 0
Index row
10 5
Remark. By examining the numbers in
increased by Rs. 10 for each unit of x
the Cj-2; row of table 2, we
mix. Thus, a positive added to the solution mix òr by Rs. 5 find that total protit can be
fhe soolution number in the Cj-Z row for each unit of x, added to
amount per unit
of X, added. On
the other hand, a negative indicates that profits can be improved by that
amount by which profits would decrease if one unit of the number in the Cj -Z row would indicate the
Sotution. Hence the optimum solution is variable heading that column were added to the
row.
reached when no positive numbers are there in
After the initial simplex tableau is set up, the
imnrovement is poSsible. The computational procedurenext is
step is to determine if the
as follows :
la) Choosing the entering variable. To
bosolution next (it must be either x1 or X2, decide which of the variables will enter
since they are
ariables at this point), we select the one with the largestthe only two non-basic
Jariable x1, has a C-Z value of Rs.10, implying th¡t each unitpositive of x1,
Ci-Z, value.
added
atition mix will contribute Rs. 10 to the overall profit. Variables X2, has ainto the
Iaue of only Rs. 5. The other two variables S and s, have zero values and C;-Z,
can add
nothing more to protit. Hence, we select xj as the variable to enter the solution mix,
and identify its column (with an arrow) as the pivot or key column.
(b) Choosing the departing variable. Since we have chosen a variable to enter
the solution mix we have also to decide which variable is to be replaced. This is done
in the following manner :
Firstly, divide each number in the quantity column, i.e., 100 and 80 by the
corresponding numbers in the key column, i.e., 4 and 5.
Secondly, select the row with the smallest, non-negative ratio as the row to the
replaced.
Here the ratios would be :
S] row: 100 hrs/4 hrs per unit = 25 units of x1,$2 row: 80 hrs/5 hrs per unit - 16 units of x1.
As the s2row has the smallest positive tatio, it is called replaced row, or the
pivot row or key row. This row will be replaced in the next solution by 16 units of
X1, Îe., the variable s (unused time) will be replaced by 16 units of xj in the next
solution.
the
Ihe number at the intersection of key row and key column is referred to as
pivot or key numnber which is 5 in the present case. COLUMN)
KEY NUMBER & KEY
TABLE 3: INITIAL SIMPLEX TABLEAU (KEY ROW,
10 5 Minimum
ratio
Basic Solution S S
values X/x,
variables
B
5 100
100 4
4
= 25 key
S
row
80
5 2 5
80
S
5
G-Z ‘ Key column " Key number
OPERATIONS RESEARCH .

6. Developing second simplex tableau. Having chosen thePROBLEMS


&
3.10 key column
tableau can be developed, providing an improved
second simplex row. For this we have to
key row, a key
(a)
for the
Compute new alues number. The key row now becomes :
by key
simply
number inthe key row Solution values
Basic variables S1
80 1 S2
C 5
0
5
10
now in the solution mix and that 16 units
that x is listed as Rs. 10 contribution per unit
It may be noted C, value is of
being produced. The with a more profitable
solotion. This will definitely
generated in the initial
provide
solution.
than solution
thxiein Rst.h1e
(derived numbers)for each remaining row.
tableau, we compute new values for the remaining row (i.e.,ToS1compet
Compute
the (b)second new values row) e a
follows

New row number Old


row number
-[°Corresponding, qumber Corfixed
in key row respondirationg
Old. row number in key column
where fxed ratio = Key number
(correspondine &.
Thus the new row s1 for second simplex tableau is derived as tollows
ratio =):
=36
100 - (80x)
= 0
4 (5x)
5 (2x) 17
5

1 (0x) =1
4
0 (1x)
These resuits are nowentered in Simplex Tableau II. The final step of the second
iteration is to introduce the effect of the objective function. This involves computing
the Z, and G-Z rows. Recallthat the Z, entry for the quantity column gives us the
gross profit for the current solution. Thé other Z, values represent the gross proit
given up by adding one unit of each variable into this new solution. The Zs are
calculated as follows :
Z, (for total profit) = Rs. 0 (36) + Rs. 10 (16) =Rs. 160
Z, (for x1 column ) = Rs. 0 (0) + Rs. 10 (1) =Rs. 10
Z, (for x2 column) = Rs. 0 () + Rs. 10 (É) = Re. 4
Z, (for s] column) = Rs. 0 (1) + Rs. 10
(0) = Rs. 0
Z, (tor $2 Ccolurnn) = Rs. 0 (-) + Rs. 10 () = Rs. 2.
It may be noted that the current profit is Rs. 160 as against Re. 0 in
i thefirst
tableau. The Ci-Z, numbers represent the net profit that will result given our
production mix, if we add one unit of each variable into the
solution.
LNEARPROGRAMMING--I|
3.11

Column
S S
forcolumn Rs. 10 Rs. 5 Re. 0 Re. 0
Z forcolumn Rs. 10 Rs. 4 Re. 0 Rs. 2
C-Z,forcolumn Re. 0 Re. 1 Re. 0 Rs. - 2)
TABLE4: SECOND SIMPLEX TABLEAU
10 5
Minimum
Basic Solution ratio
C variables, B values, Xp X S S

S 36 -4 36
1
17/5 + key
row

16 1 1 16
10
2/5
160 10 4
1 -2

T Key column Key number


7. Developing third simplex tableau. The presence of a positive number under the
rocolumn of the Cj-Z, row of the second tableau indicates that the total
contribution can be increased further. Hence the process used todevelop the second
solution must now be repeated to obtain a third solution. Accordingly, we find that
(a) The variable x2 will enter the solution next by virtue of the fact that its C;-2;
value (+1) is the largest (and only) positive number in the row. This means that for
every unit of x that we produce, the objective function will increase in value by
Re. 1. The X, column is the new key column.
(b) Again, each number in the quantity column is divided by its corresponding
number in the x> Column.
36 180 16
For the S1 row : 17/5 17 and for the x1 row : 2/5 = 40.
The s row has the smallest ratio, meaning that variables s1 will leave the basis
and be replaced by X2. The new key row, key column and key number are shown in
table 5.
(c) The key row is replaced by dividing every number in it by key number, i.e.,
Ihe entire new x> row will look like this :
C Basic variables Solution values X2 S S
5 180 0 1 5
17
4
17
It will be placed in the newsimplex tableau in the same row position that sË was
in before.
(a) The new row X, for Tableau IlIl is derived as follows:
New row
Oldrow -Corresponding number Corresponding fixed ratio number
number in key row
(36 x ) 200
7
16
=1
1 (0xý)
=0
(17x4)
0 (1xp) =
17

- (-x)
OPERATIONS LEMS
third
3.12
row will appear
in the tableau in the
Hence,the newxË
(e) Solution S
position Basic values
CB variables
(B) 5
180
17
5 200 1
5
17
1
thethird tableau can be
Z, rows for
calculatet
10
Z, and C;-
Finally, the
) earlier.
discussed follows :
unit of product appears as
The net profit per Column
S
S,
5
10
5 5
10
Z for column
5
30
G-Z colunn -17

third iteration of the simplex method are summarid.


All results for the
that since every numberin the tableau's C-Z, row is zero or negative
table 5. Note has been attained,
an optimum solution SIMPLEX TABLEAU
TABLE 5:THIRD
10 5 0
C
Basic Solution S S
Variables values
B XB
4
180 0 1 17
5 17
200
2 5
1 17
10 17
5 30
2900 10 5 17
17
17 30
5
17 17

200 180
Hence the optimum solution is : x1 = 17 X2 = 17

Sj =0, slack hours in the machine M1


$p = 0, slack hours in the machine M2
Profit = Rs. 900
17 for the optimum solution. through the
Remarks 1. Since it is always possible to make an arithmatical error when one s goingcanbedone
This
numerous simplex steps and iterations, it is always advisable to verify the final solution.
as follows :
First constraint : 4x, +5x, s 100 (Machine M, hours)
Or 4(200/17) +5(180/17) s100
or 100 s 100
Second constraint: 5x +3x s80 (Machine M, hours)
Or 5(200/17) + 2(180/17) <80
or 80 < 80
LINEARPROGRAMMING-| 3.13

Objectivefunction; Profit =19x1 +5x2


- 10x7+5x
200 100
17
- Rs. 900
Interpretationoftheefinal simplex table:
2 The simplextable helps to predict the effect of changes in the input factors (resources) and the profit
marginwithout drawingthe table afresh.
(a) Opportunity cost. The terms in Z; indicate the opportunity cost. If we do not utilize one unit of
loss of profit is Rs. 10 and non-utilization of one unit of x) cost Rs. 5.
the
A1,
Ib) Shadow cost. In our previous solution, the row Z; gives

2900
17 - 10x1 +5x, + 51+2
For maximum value of Z we put s, = 0 and s, = 0sothat Z = 0 The coelficient of non-basic variables,
17
s1 and $o is called 'shadow cost'. They represent the decrease in the optimum value of the
ie.,of
objectivefunction resulting from a unit increase in a non-basic variable.

Rroblem 3-2. Acompany sells two different products Aand B. The selling price
and incremental cost information are as follows:
Product A Product B
Selling Price Rs. 60 Rs, 40
Variable Cost Rs. 30 Rs. 10

Thetwo products are produced in acommon production process and are sold in
tuo different markets. The production process has a capacity of 30,000 man hours.
ttakes three hours to produce aunit of Aand one hour to produce a unit of B. The
market has been surueyed and the company officials feel that the maximum number
of units of A that can be sold is 6,000 and maximum for B is 12,000 units. Subject
to these limitations two products can be sold in any combination. Formulate the
Droblem as LP model and solue to maximize contribution. (Use simplex method)
Solution Steps 1. Formulation of the LP model. Contribution of the product A=
Rs. (60 - 30) = Rs. 30. Contribution of the product B= Rs. (40 - 10) = Rs. 30. The
complete linear programming model can be stated as follows :
Maximize (total contribution) Z = 30x1 + 30x2
subject to the constraints
3x1 + X s30,000
X1 S6,000
xy S 12,000
x1 20, X 20
Wnere x and x denote the number of units of products A and B to be manufactured
Tespectively.
2. Converting constraints to equations. The constraints for the given problem
can now be rewritten with slack variables to form the equalities as given below :
3x1 + X2t S1 30,000
+ S2 =6,000
+ S3 = 12,000
xË 20, x2 20, s1 20, s2 0, s3 >0
OPERATIONS RESEAH

3.14
represent unused
resources, their
variablesin the objective
pHcontribution
UBLEMS OUIO in the
objectivefunction
get
variables
Since slack zero. Including
is
these slack

30x, +0sj +0s + Os3


MaximizeZ= 30x + logic of the simplex method is
we, funcion,
the Initial Solution. The feasible solution region can
of the
on the basqiveed
indicatizer0,nguniqe
up
3. Set the extreme points solution at the origin
with a
fact that only
solution. The search starts the values of decision variables are
therefore
tha
optimum producedand not producing anything, obviously we i.e
= 12,000 We note are leth
nothing can be When we are
x3 =0. and s3
and
x1 =0, X =0capacity of sj = 30,000, s2
=6,000
variables S1,s2 and s3) with the that
with unused
has three
variables
current solution three variables(decision
(slack
variables X1, X2, and x3) with zero
basic variables. values
withVariables
nonzero
solution values and values are called
solution
Variable with non-zero
non-basic variables.
values arecalled OF INITIAL
SIMPLEX TABLEAU
TABLE l : PARTS
Slack variables
Basic Values Real variables
Profit basic column
per unit variables of column
column column variables
column
Solution 30
+Profit
Basic 30
variables values x(= s) K=s) x% =SVariable rou
B b =X)
C 1 Rows
30,000 3
0 S illustrating
constraint
S 6,000 equations
1 (coefficients
12,000 only
Body matrix Identity matrix
consisting of consisting of coefficients
coefficients of of slack variables
real product
variables

Starting from left-hand side of the above table the column C indicates the contribution
per unit for the slack variables s1, S2 and s3. Since the profits are not made on unused time,
the contribution per unit for S1, S2 and s3 is shown as zero.
The number in the Z, row under each variable represents the total contribution of
outgoing profitZ,when
basic variable. valuesonein unit of avariable
various non-basiccolumns
variableareis calculated
introducedbyintomultiplying
the basisthe
inplace of a
coefficients
of basic variables in the Cg column with coefficients (a) in each variable column and then add
up the products so obtained. For example Z, value in the x;
column will be :
Z = 0(C,of s,) x 3 (coefficient of xj
in s, row, i.e., a,j)
+ 0(C, ofs) x 1(coefficient of x1 in s,
+ 0C; of s,) x 0
row, i.e., a,i)
(coefficient in xjin s, row, i.e., azi)
=0x3+0x1+0 x0 =0
Following the same procedure Z, for all the other variable columns will be
computed as shown in the table 2.
The numbers in Cj -2, (Index or Net Evaluation Row) now represent the net
profit, i.e., profit gained minus the profit given up that will result from introducing1
LINEARPROGRAMMING|
3.15
basic
unit
of each
the
variable into the solution and can be
Z.total
at computed
subtracting for each column from the C, by simply
variable's
's column. For example, Cj- Z value in value the very top of that
30and so on. The values of C- Z are always the x column will be C,-Z, = 30 -0
-
of
zero under the basic variables.
The value the objective function in the
nmultiplyingthe Cg column with the current solution is obtained by
thenadding these
products, i.e., Z = corresponding
30 x 0 + 30 x elements in the X¡ column and
0 = 0.
TABLE 2:INITIAL SIMPLEX
Basic Solution
TABLEAU COMPLETED
30 30
C variables values
B bex S
30,000 3 1
6,000
0
0 S 12,000
Z=0
30 30

Remark By examining the numbers in the C;-Z;


he increased by Rs. 30 for each unit of x, added to (index
or NER) row of table 2, we find that profit
the solution mix or by Rs. 30 for each unit of x,
dod to the solution mix. Thus a positive number in the C,- Z; row
a unit of the variable heading that column were indicates that profits can be improved
added to the solution.
4 Test for Feasibility. After the initial simplex table is set
up, the next step is to
determine whether any improvement is possible. The computational procedure is as
follows :
(a) Choosing the entering variable. Since the net increase of profit per unit of
each product is represented by numbers in C;- Z, row, the product to be introduced
first will be largest positive value of C;-2;, It may be noticed in table 2 that the
largest positive value of C-Z in index row is 30 under the column x and x, both.
This indicates that XT units be produced first in order to increase total profit at the
taster rate, since each unit of xp added into the solution mix will contribute Rs. 30 to
the overal profit. The variable x which is to be brought into the basis in the second
simplex table is known as entering variable and the column corresponding to
entering variabte 1s termed as key or pivot column.
(b)Choosing the departing variable. Since we have decided to enter one variable
a5 the basic variable into the solution basis, we must replace one of the existing
Dasic variables to depart from the solution basis. The variable to depart is identified
Dy torming the ratios of solution values to physical rates of substitution of entering
Variable, i.e.,we divide each number in the quantity column, i.e., 30,000; 6,000 and
l,000by the corresponding numbers in the key column, i.e., 1, 0and 1.
Thus for S1 row, ratio 30,000/1 30,000
for s row, ratio
and for Sa rOW, ratio 12,000/1i = 12,000

The one with the minimum positive ratio (s2) represents the variable to depart
Irom the solution basis. Thus s, is the departing variable. The row coresponding to
he departing variable is caled key row or the pivot row or the
replaced row. The
key element and
on the intersection of key row and key column is called
iselement
denoted by makinga () or circle in the simplex table.
OPERATIONS RESEARCH PROBLEMS

3-16 NUMBDD
TABLEAU, KEY ROW, KEY KEY COUMN
SIMPLEX
TABLE 3:INITIAL 30 0 0
30
Solution
Basic
variables
values
beX
S S S Rallo
0
3

0
30,000
6,000
1 0
0
30,000/|
S
12,000
12,000/1 6
Z=0
30
Z 30
Key row
C-Z
Key column
5. Developing second simplex table. In order to obtain the next solution, edig
follow theforfollowing steps :
refer(a)
to table 3
Compute and
new values the key row. To revise the key row, divide all value
in the key row(s2) bythe value of the key element (i.e., 1) and replace departing
variable ($2) bythe entering variable(x1). Also replace the new values in C; columm
so obtained at the appropriate places. Ih
accordingly. Put all other values
example, this new row becomes : 1
1
12,000
(derived rows. For nk.
numbers) jor each remainingformula
(b) Compute new values .
can be obtained by using the
non-key rows, say S1 row, new values
New row
[Old row Coresponding number Corresponding
in key row fixed ratio |= number
number
= 18,000
30,000 - (12,000 x 1)
=

3 - (0 x 1)
= 0
1 - (1x1)
= 1
1- (0 x 1)
= 0
0- (0x 1)
= 0
0 - (x 1)
way as discussed
The values of Cand (C;-Z) rows are calculated in the same values o
earlier. The value of the objective function is calculated by substituting the
the variables in the objective function.
The new revised improved solution is shown in table 4 below :
TABLE 4: SECOND SIMPLEX TABLEAU
Basic Solution 30 30
variables values
Ratio
B b-X¡)
(Xg/x)
18,000 3
18,000/3 = 6,000
6,000 1 6,000/l¢
30 12,000
Z= 0 30 0 0 30
Key row
3,60,000 30
C-Z -30

"Key element ‘Key column


LINEARPROGRAMMING
317
in the same
6. Proceeding manner, we notice that variable x will enter into the
solution varlable s1
arnd will depart. The key element is The next improved
below in table 5. solution
TABLE5: THIRD SIMPLEX TABLEAU
Bastc Solutlon
variables values 30 0
S S
S
6,000 -3
12,000
Z= 30 30 30 30
5,40,000
-30 -30
Since all entries in the index row (C- 2) row of the
kie indicates no sign of further improvement in the above table are negative,
Tan optimum solution has been arrived at and the contribution, Thus it follows
x1 =6,000; x2 = 12,000 and max Z Rs.
solution values are given by :
5,40,000.
Interpretation of the Final Simplex Tableau : The simplex table helps to predict the effect
af changes in the input tactors (resources) and the profit margin without
afresh. drawing the table
lo) Opportunity cost. the terms in Z, indicate the opportunity cost. If we
ne unit ofx1, the loss of protit is Rs. 30 and non-utilisation do not utilize
of one unit of x costs Rs. 30.
(b) Shadow cost or price. In our previous solution, the row Z gives :
5,40,000 = 30x1 + 30x2 + 30s1 + 30s2
For maximum value of Z, we put s1 = 0and s2 =0so that Z = 540,000. The coefficient
non-basic variables, i.e., of s1, S2 and s3 is called shadow cost. They
in the optimum value of the objective function resulting from a unit represent
the decrease
increase in a non-basic
variab)é.
Problem 3-3, Acompany produces three productsA, Band C. These products
require three ores O1, O, and O3. The maximum quantities of the ores O, O2 and O3
available are 22 tonnes. 14, tonnes and 14 tonnes respectively. For one tonne of
each of these products, the ore requirements are
Profit per ton
(in Rs. 'O00)
A
3 1
B 2 2 4
C 3 3

How many tonnes of each product A, B and C should companyproduce to


maximize the profit.
Solution. The appropriate mathematical formulation of the given problem is:
Maximize (total profit) Z = x1 + 4x2 + 5x3
subject to the constraints
3x1 + 3x3 S 22
X1 + 2x2 + 3x3 s 14
3x1 + 2x2 s 14
X1, X2, X32 0
Introducing slack variables s1, s2, $3 2 0, the LP problem is written in the
standard form as
Maximize Z = X1 + 4x2 + 5x3 + Os1 + Os2 + Os3
subject to constraints
3x + 3xg + Sj = 22, xË + 2x + 3x3 + sS2 = 14
OPERATIONS RESEARCH:
PROBLEMS &SOT
3.18 20.
S3= 14; X1, X2,X3, S1, S2,S
3x,+ 2x +solution is :
An initial basic
feasible
22, s) = 14, sz = 14
=0,S1 = follows .
X} = X)= X3 starting simplex table Ias
TABLEenables
I : INITIALusSIMPLEX the WITH KEY ROW, KEY COLUMN AND KEY ELEMEN
to set upTABLE
This
5

Basic
C
Solution
1 4

S S, S3
Minratioimum
variables values
0 22
0 3
3 3
22
0 S
14
14 3
S 1 2
14
0
0

1 4 51
(Index row) G-Z
find that -5 is the most negative shadow m
Observing the index row, we
the basis. Now to determine the departing variable. for
we compare
the noes
Thus x3 will enter column and obtain the ratios
column with X3
the current solution
are
components of the key columnn which
14
the least ratio which corresponds to the second row, S, will leave the
Since 3 is update
Thus x* enters and s, leaves the basis. The key element is 3. We now
basis. variables sS1, X3 and s3 by performing a set of
the current simplex table with new basic
elementary row operations.
(old)-3 R (new), R3 (new) ’ Rg (old)
R,(new) ’ R, (old/3;R, (new)’R
obtained :
The followingfirst iterated table is
TABLE II:SECOND SIMPLEX TABLEAU
5 Minimum
1 4
C ratio
S S S
B X
2 -1
0 S
14/3
14 1 2
5 3
1 3 2/3
3 3
14/2
14 3 2*

5 10
3
5

(Index row) G-Z 5

from 0to
It may be noted that the value of objective function has been improved isnot
10 Hawever, since there is a negative shadow price, the current solution
14/3 14. 4
optimum. Clearly, x2 must enter the basis. Further, since min. 2/32
choose53
corresponds to x3, S3, either of them and be removed from the basis. We
to leave the basis. Thus x> is the key column and s3 represents the key row.
LINEAR PROGRAMMING--I| 3.19

Applyingthe elementary row operations :


(new)-’ R3 (old)/2, R1 (new) ’ RË (old) +2R3 (new), Ra(new) ’ R2 (old) - Rg
2 (new)
The following iterated table is obtained :
TABLE III :THIRD SIMPLEX TABLEAU
4 5
B
X S S S
S 22 5 0 -1 1
2
1
3
7 3
4
2 2

5 1
3 4 5 3
5
(index row) G- Z 3

Since all entries in the C;- Z, row of third simplex tableau are either zero or
negative,optimum solution has been obtained, and the maximum value of Z occurs
when x =7and xj =X3 = 0.
Thus, for a maximum profit of Rs. 28,000, the company should produce 7
tonnes of product B and none of A or C.
Problem 3-4, Anelectronics firm is undecided as to the most profitable mix for its
products. The products now manufactured are transistors, resistors and carbon
ubes with a profit (per 100 units) of Rs. 10, Rs. 6 and Rs. 4 respectively. To
produce shipment of transistors containing 100 units requires 1 hour of
engineering, 10 hours of direct labour and 2 hours of administration service. To
produce 100 resistors are required 1 hour, 4 hours and 2 hours of engineering,
direct labour and administration time respectively. To produce one shipment of the
tubes (100 units) requires 1 hour of engineering, 5 hours of direct labour and 6
hours of administration. There are 100 hours of engineering services available, 600
hours of direct labour and 300 hours of administration. What is the most profitable
mix?
Solution. Let x1, X and x3 denote the number of units of transistors, resistors
and carbon tubes respectively. For the sake of convenience, we tabulate the data in
the following manner :
Products
Available hours
Transis tors Resistors Carbon tubes
1 100
Engineering 4 5 600
Labour
2 6 300
Administration 2
Rs. 6 Rs. 4
Profit per 100 units Rs. 10

Maximize Z = 10x1 + 6x2 + 4x3


subject to the constraints
X] + X2+ x2 S l00
10x1 + 4x + 5x, s 600
2x1 + 2x, + 6x3 S 300
X], X2, X3 2 0
OPERATIONS RESEARCH:PROBLEMS &
3-20 The first step in the simplex method is to convert the inequalities
adding a slack
byvariables,the LP variable
ofthe department). After adding
equalities. is done
This the slack problem(unused
is writtennin he
into
restrictions)
capacthy
standard form as :
+ 4x3 + Os; + Os2 + Os3
Maximize Z = 10x;+ 6xp
constraints
subject to the
t X2 + X3 + Sj = 100
X] 600
S) =
10x + 4xX + 5x; +
s3 = 300
2x + 2x2 + 6xg + solution, i.e., it
begins with azero the
The simplex method always
This enables steps in the starts at the
subjesolctution
production whatsoever. to
point of no appropriate quantity of each item to be produced, to th e
determine the given constraints.
objective function and the not produced, then the600 unused
In other words, if x, X, and x3 are
and s3 will be 100, and
300capacity
of
the three departments as
given by S1, S2 hours
respectively.
solution is :
An initial basic feasible
X3 = 0and sj = 100, S, = 600, s; = 300.
XË = X)= SIMPLEX TABLEAU
TABLE I:INITIAL
10 6 4
C Minimum
Basic Solution
variables values
o-| S S ratio

B
1 1 10
100 1
4 5 1 60
0 S 600
1 150
2 6
0 S 300
0

(ndex row) G-Z 10‘ 6 4

Applying following row operations to get a new solution by entering variable x


:
into the basis and removing the variable s, from the basis
Rz (new) ’ R2 (old)/10, R, (new)’ R, (old) - R2 (new)
and Rg (new) - R3 (old) - 2R, (new)
The new solution is shown in table 2 below:
TABLE 2:SECOND SIMPLEX TABLEAU
0 Minimum
10 6 4
ratio
B X S S

40 6* 5 67 -
S 1
10
4 1 150
10 60 5 0
10 10 10
150
0 Sg 180 12 5
2
1
10 10
10 4 5 1
(Undex row) G-Z 0 21 -1 -1
variable X2
Applying following row operations to get a new solution by entering
into the basis and removing the variable s from the basis :
LNEARPROGRAMMING 3.21

R(nneew) -’ R (old)/ Rz (new) -’ Rg (old) - R(new),


Rg (new) R3 (old) - R(new)
and
solution is shown in table 3below :
The neW
TABLE 3:THIRD SIMPLEX TABLEAU
10 6 4

Basic Solution
variables values
S S
B
400 5 10
6 1
6
100 2
3 6 6
10
S3 100 4 2
N 20
10 6 10
3
10

As there are no positive entries in C;-Z row, no further improvement is


nnsible, and the optimum solution has now been obtained which given by:
400 2200
X1 3X 6 and x3 =0; maximum Z = 3
Problem 3-5. A manager produces three items A, Band C. He has the possibility
of applying two strategies--produce all the three items or any two of them.
Products A and C pass through shops I and I, whereas B is further processed in
shop ll.,Each shop has limited available hours. Hours auailable in shop I, II and II are
162hours, 189 hours and 5 hours respectively. Profit per unit from A, Band Cis
Rs. 27, Rs. 29 and Rs. 25 respectively. The following table gives the processing
time of different items in different shops.
Items
Shops A B
27 12 12
27 15 25
3
Find the optimum production of A, Band Cso as to maximize profit.
Solution. The appropriate mathematical formulation of the given problem is:
Maximize Z= 27x1 + 29x, + 25x3
subject to the constraints
27x1 + 12x + 12x3 S 162
27x1 + 15x + 25x3 S 189
3x2 s5
X], X2, X3 2 0
By introducing slack variables s1 2 0, $2 2 0 and s3 0, the LP problem in
standard form becomes :
Maximize Z=27x1 +29x, + 25xg + 0.s1 + 0.s2 + 0.s3
subject to the constraints
27x, + 12x, + 12x3 + S1 = 162
27xj + 15x, + 25x3 + S2 = 189
3x2 + S3 = 5
X], X2, X3, S1, S2, S3 2 0
OPERATIONS RESEARCH:
3.22
An obvious initial basic feasible solution is obtained bu setting x1 =
PROBLEMS &
and s3 = 5 with maximunm Z-0.
Thus S] = 162, S) = 189, SIMPLEX TABLEAU
TABLE 1:INITIAL
29 25 0
Solution 27
C’ Basic
variables values S S Minratimiumo
12 12 0 81
162 27 12
6
27 15 25 1 63
S 189
5
5 0
S

27 29‘ 25
0
0
5key roR
(Index Row) C-Z
TABLE 2:SECOND SIMPLEX TABLEAU
27 29 25
(Introduce x, and drop sÙ
Basic Solution
variables
B
values S, S Minratio
imum
142 12 1 0 -4
6 key row
27 25 1 -5
0 S, 164 14
27
5 1
29 3
29
145 0 29 0
3
3
29
271 25 0 0
3
(ndex Row) C-Z
TABLE 3:THIRD SIMPLEX TABLEAU (Introduce x, and drop s)
Basic Solution 27 29 25
variables Ualues Minimum
S S3 ratio
B
27 142 1
12 71
27 0 6
27 27 27
22 0 22
-1 1 -1 13 +key row
29

571
27 29 12

(Undex Row) C-Z 0 13‘ -1 17

TABLE 4: FOURTH SIMPLEX TABLEAU (Untroduce x, and drop s)


Basic Solution 0
29 25
varlables values
B S
S
27 1582
X
351 25 12 35
25 22 351 351
13 0 0
13 13 13
29
3 1

Z 8291 182
27 29 39
25
182
(ndex Row) C-Z 0 -1 39
NEARPROGRAMMING- 3-23

Sinceall
the numbers in the C;-Z, row are either negative or zero, the optimum
t othe given problem has been obtained and is given by :
solution
1582 5 22 8281
X= X2 = and x3 = with maximum profit of
and
has 100 kg. offA, 180kg. of Bbasic
Problem 3-6. APharmaceutical Company
of Cqvailable per month. They can use these materials to make three
pharmaceutical
120 kg. products namely 5-10-5,55-10 and 20--5-10, where the
respectively
numbersin each case represent the percentage by weight of A, Band C
products. The cost of these raw materials are given below:
each ofthe
ne Cost per kg. (Rs.)
Ingredient
A 80
B 20
50
Inert ingredients 20

of these products are Rs. 40-5, Rs. 43 and Rs. 45 per kg.
Selling price
capacity restriction of the company for the product 5-105;
respectively. There is how much of
produce more than 30 kg per month. Determine
soas they cannot
should produce in order to maximize their monthly profit.
Poch of the products they
be denoted by P,, Pa and P3.
Solution. Let the products to be manufactured
Ingrediernts
Inert
A B
Product 5% 80%
5% 10%
P 10% 80%
5% 5%
10% 65%
P 5%
20% 20
P% 50
20
80
Cost per kg.
Rs. 24-50/kg.
5% x 80 + 10% x 20+ 5% x 50 + 80% x 20 =
Cost of P =
+ 80% × 20= Rs. 26/ka.
5% x 80 + 5% x 20 + 10% x 50
Cost of P = 20 = Rs. 35/kg.
20% x 80 + 5% x 20 + 10% x 50 + 65% x
Cost of P, =
40-50 - 24-50 = Rs. 16
Margin ofP, = = Rs. 17
Margin of P = 43- 26 = Rs. 10
Margin of P3 = 45-35 formulated in mathematical form as follows :
Now, the problem can be
Maximize Z= 16x1 + 17x2 + 10x3
subject to the constraints
s 100
....(1)
s 2000
or
4x3
1 s 180
...(2)
S3600
X3
Or 2x1 + S 120
...(3)
s 2400
2x3 .(4)
X] + 2x + s 30
Or X1
...(5)
2 0
X1, X2,X3 products to be
quantity in kg. of the three
denote the
where x1 X2, X3
manufactured.
OPERATIONS RESEARCH:
3.24 PROBLEMS &

raintsSOLUTab0.ION
S1, $2, $3 and s4 for the four
Introducing the slack variablestableau constra
the initial simplex as follows :
we shall prepare TABLEAU
TABLE 1:INITIAL SIMPLEX
17 10 0
Basic Solution 16

Minratimiuom
variables values S S, S S4
B
4
0
0
S

S
2000
3600 2
2
1 1
2000|3600|11
Se
2400
30
16 17 10 0
2400/Not. 2+def
16 17t 10

TABLE 2:SECOND SIMPLEX TABLEAU (Introduces x, and

C Basic Solution 16 17 10 replace s)


variables
B
values
S S3 S4 Minratloimum
0 S, 800 3 800
3
1/2
S 2400 2
1 2400
1
3/2
17 1200 1 1200
2 2
1/2
30 1
30/14
17
17 17
2
17
G-Z -7
2

TABLE 3: THIRD SIMPLEX TABLEAU (Introduce x, and drop s)


Basic Solution 16 17 10
variables values
B S S S S

S, 785
3 1 0
S 2355
0
17 1185
1
16 30 2
1 1
0
20,625 16 17 17
17 2
2
G 0 -7 17 15
2
Since all the numbers in the C;- Z, row are either zero or negative, optimum
solution has been obtained and is given by :
X| = 30kg ; x2 = 1185 kg. and
maximum Z = Rs.
Problem 3-7. Acompany produces two products Aand B, each20,625.
of which requires
three types of processing. The length of time for processing each unit and the profit
per unit are given in the following table:
Product A Product B Available capacity
(hr/unit) per day (hr.)
Process I 120 (hr/unit)
Process II 120 8.400
30
60 3,000
LNEARPROGRAMMING-I| 3-25

ProcessI l 80 40 4,800
(Rs.) 50 70
Profitperunit
How many
units of each product should the company produce per dayin order
maximize the profit ?
(o
Solution. Let x, be the number of units of product Aand x be number
of units of
company. The objective function and the constraints of
product B, produced by the
given problem, as per the givern data are as follows :
the
Maximize Z=50x; + 70x,
constraints
subject to the
120x1 + 120x2 S 8400 or X] + X)s 70
30x1 + 60x2 S 3000 Or X] + 2xS 100
80x1 + 40x, S 4800 Or 2xË + X2s 120
X1, X2 2 0
and ss 0so as to
Introducing non-negative slack variables s1 2 0, s>0 programming
cnnuert the constraint inequations into equations, the given linear
is:
Droblem in its standard form
Maximize 2 = 50x1 + 70x + 0.s1 + 0.s
subject to the constraints
X1 + X2 + Sj = 70
X1 + Zx2 + Sy = 100
2x1 + x2 + S3 = 120
X], X2, S1, S2, S3 20
An initial basic feasible solution is :
X1 = X2=0, S1 70,s, = 100, and s3 = 120.
components involved in the algorithm
Starting with this solution, the various
are shown below :
SIMPLEX TABLEAU I, II & III
50 70 Minimum
Basic Solution
ratio
variables values
B X 70
0
70 1
S 100/2
2* 0
1 120
100
120 2
S
0 0
, enters & Z
70‘
|,leaves C-Z 50
1
S 20 2
R8 & 1
70 50
2 70/
S 70
70 35
|X enters & 3500 35 0
-35
s, leaves 15‘
-1 0
2
50 X 40 1 -1 1
70 30 -3 1

S 10 20 0
70 -30
4100 50 - 20
0 ( -30
OPERATIONS RESEARCH:
3-26
negative
PROBLEMS &
last row are either or zer0,
Since all the numbers in the
problem has been obtained and is the
feasible solution to the given 4100. Hence the company will produce
by given opiru
Z= Rs.
A2 = 30 with maximum 40
product Problem
respectively.
of B produces three products P,. Pz and P3 from
30unitsAcompany
Aand 3-8. unts
product P1 requires one unit of A,
materials A, Band labour L. One unit of required 2 units of Aand 3
Band 2 units of L.
units of L,haswhile
company
A unit of product
one availability
a daily
needs
P2
units of A, 6 units of Band 4
B
unit of Pa of 8 units ofA, 12 units of Band 12 units ofof 3
2 and
o
L. The
each, units
units
products is Rs.
further known that the unit contribution margin for the as a 3, 2 5
respectively for P, P and P, Formulate this problem linear .and
problem, and then solve it to determine the optimum product mix.
Is the solution obtained by you unique ?ldentify an alternate optimum
programming
any. Also, obtain the shadow prices of the resources.
Solution. Let x, x, and x3 denote the number of units of products
solution,
and P; respectively. Introducing three slack variables s1 , S2 and s3, the proble
be restated as follows :
Maximize Z=3x +2x +5x3 + Os + Os2 + 0s,
subject to the constraints
X1 + 2x + 2x3 + S1 = 8
3x + 2x, + 6xg + s = 12
2x1 + 3x2 + 4x3 + S3 = 12
X], X2, X3, S1, S2, S3 2 0.
We shall now solve the problem by writing all the relevant simplex tables one
after another till the optimum solution is obtained.
SIMPLEX TABLEAU 1, II &I
Basic Solution 3 5
G’ Ratio
variables values
S S
S 1 2 2 0 4
12 3 2
+2
12 2 3 4 3
X, enters 0
S, leaves 3 0
2
51
S 0 4
1
5
2
3 1
5
6 +4
S 4 0 2
X enters 10 5
2 5 0
&x, leaves G-Z 0
1
3 0 5

S, 4 0
3
3 2
4
2 6
0
Sa 4
2 12
12 3 5
2 6
0
LINEAR PROGRAMMING-
3-27
Since all the entries in C, -z. row are either negative or
solution has been obtained and the maximum value of Z is 12 which
zero, an optimum
4, x, -0 and x3 = 0. occurs when
(ii) Since x2 is a
non-basic
variable and the corresponding C, -Z, entry in
the
final sinmplex tableau is 0, the optimum solution obtained is not unique. To obtain an
alternate optimum solution, we pertorm one more iteration.
variable, we obtain the following tableau : Taking x, as an entering
Basic Solution 3 2 5
G’ variables values
S S S3
S 4
5
3
12
5 2
3
5 5
12 3
5 5
12 3 2 6 1
- 1 -1

12
The optimum solution is : x1 .x2=5 and x3 = 0 and the maximum value of Z
is 12, as before. It may be noted that the entries under the columns sl, S2, $3 in the
C,-Z, rowof the final tableau are 0, -1, 0. Their absolute values, viz., Õ, 1, 0 are
known as the shadow prices of the resources. Hence the shadow prices of the
resources are 0, 1 and 0respectively.
Problem 3-9. A farmer has 1,000 acres of land on which he can grow corn,
wheat or soyabeans. Each acre of corn costs Rs. 100 for preparation, requires 7
man-days of work and yield aprofit of Rs. 30. An acre of wheat cost Rs. 120 to
prepare, requires 10 man-days of work and yields a profit of Rs. 40. An acretof
soyabeans cost Rs. 70 to prepare, requires 8 man-days of work and yields a profit of
Rs. 20. If the farmer has Rs. 1,00, 000for preparation and can count on 8,000 man
days of work, how many acres should be allocated to each crop to maximize profit?
(Jammu Univ. M.BA, Feb. 1996)
Solution. The appropriate mathematical formulation of the above problem is as
follows :
Maximize Z = 30x1 + 40x2 + 20x3
subject to the constraints
10x1 + 12x, + 7x3 S 10,000, 7x1 + 10x2 + 8x3 S 8,000
X], X2, X3 2 0
X1 + X2+ x3 S 1,000;
where X1, X2 and x3 designate the acreage of corn, wheat and soyabeans
respectively.
Introducingnon-negative slack variables s1 0, s2 > 0 and s3 > 0 so as to
Convert the constraint inequations into equations, the given LP problem in its
standard form becomes :
Maximize Z = 30x1 + 40x2 + 20x3 + Os1 + Osg + 0s3
subject to the constraints
10x1 + 12x2 + 7x3 + S1 = 10,000
7x1 + 10x2 + 8x3 + S = 8,000
X1 + X2 + X3 + S3 = 1,000
X1, X2, X3, S1, S2, S3 2 0
3-28 OPERATIONS RESEARCH: PROBLEMS &
An initial basic feasible solution is:
X1 =X2 = X3 = 0; Sj = 10,000; s2 = 8,000, and s3 = 1,000.
SOLUTIONG
Starting with this solution, the various computations involved in the
are shown below :

G’ Basic
SIMPLEX TABLEAUI, II& III
Solution 30 40 20
algorith,
variables values

S 10,000 10 7
S S, S3 MinTatlo
imum
S
S3
8,000
1000
7
1 2
8
1
1 10,800000/-12
X, enters and 30 20
1000
S, leaves 40 ‘

S1 400 16/10 -26/10 1 -12/10


40
X 800 7/10 1 8/10 1/10
250 -
S3 200 3/ 10 2/10 -1/10
8000/7
X, enters and 2000/3
S, leaes G-Z 16t 4 -2

250 -26/16 10/16 -12/16


40 625
31/16 -7/16 10/16
S3 125 11/16 -3/16 1/8 1

G-Z -35/4 -5/4 -5/2


Since all the entries in the C;- Z; row are either negative or zero, the
feasible solution to the given LP problem has been optimum
obtained.
wheat and soyabeans are 250, 625 and 0 respectively in Hence acreage for com,
maximum profit of Rs. 32,500. order to achieve a
Problem 3-10. By means of some sharp bargaining with the union and
subsequent reduction of union "make-work" restrictions in his former contract ,a
small paper towel manufacturer has created some spare capacity
main production departments : Cutting, folding and packaging. inForeach the
of his three
identification, three different sizes of paper towel are called products A,purpose B and
of
C
Owing to its small size, the company can sell in the market, all that it can
a constant price. Management is inclined to be produce at
expand production facilities at this time, althoughconservative and does not wish to
present spare capacity.
they do wish to utilise fully the
The paper toweling is received from another manufacturer in large rolls. Thes
rolls are subsequently cut, folded and packaqged in three sizes. The pertinen
manufacturing and profit information for each size of paper towel is summarized us
below :
Department Size Constraint for time
A B C period per week
Cutting 10.7 5-0 2.0 2705
Folding 5.4 10.0 4-0 2210
Packaging
Profit contribution
0.7 1.0 2-0 445
per unit (in Rs.) 10-0 15-0 20-0
(i) Formulate the problem using simplex method.
(ii) Find the optimum program
(iüi) Find out the optimum profit.
NEAR PROGRAMMING-I|
3-29
.Giue the
thefinaltable. economic interpretation of the entries in the net evaluation row of
Solution. The
appropriate mathematical formulation of the given
Maximize Z= 10x1 +15x +20x3 problem is:
subject to the constraints
10-7x1 + 5x+ 2x3 S 2705
5.4x1 + 10x, + 4x3 s2210
0-7x1 + X2+ 2x3 s445
x 20,(i = 1, 2,3).
Introducing non-negative slack variables s 20, s, 20 and s3 20 so as to
A constraint inequations into convert
its standard form is : equations, the given linear programming problem in
Maximize Z = 10xj + 15x, + 20xg + Os +0s, + 0s3
subject to the constraints
10-7xË + 5x2 +2x3 + S1 =2705
5:4x1 + 10x, + 4X3 + S) = 2210
0-7x1 + X2 + Zx3 +S3 = 445
X1, X2, X3, S1, S2, S3 20.
An initial basic feasible solution is :
X1 = X) = X3 =0, s1 = 2705, Sp = 2210 and s3 =445.
Starting with this solution, the various components involved in the algorithm
are shown below:
SIMPLEX TABLEAU I, II III &IV
C Basic Solution 10 15 20
Minimum
Cpl variables values S S S ratio

0 S 2705 10-7 5 2 1 2705/2


0 2210 54 10 1 2210/4
445 07 1 445/2
0
X enters 0
&s, leaves 10 15 20 ‘ 0
G-4
10 4 -1 2260/4
0 S 2260
1320 4 -21 1320/8
20 222-5 7/20
1
1 2 222.5/5
10 20 10
enters , 4450 7
5‘ - 10
&s, leaves C,-Z 3
1 1600/8
0 S, 100 8
15 165 1 4
5
20 140 10
16 8 140/ 10
5275 15 20 5/8 35/4
enters 19/2 T
-5/8 - 35/4
&s leaues G-4
0
16
10 200 5
15 65 16 32
1 9 5
1 80 160 8
20 120
15 20 1/16 19/32 35/4
5375 10 -1/16 - 19/32 -35/4
Z,
OPERATIONS RESEARCH:PROBLEMS
3-30

program x 200
is at when 200, x2= 65 and X3 = 120.
present
(i) spare
The optimum
capacity is utilised fully units of towel A 65 units of

and 120 units of towel C


are manufactured. Thus, he
Rs. 5375.
(i) The optimum profit is evaluation (or C-Z)
(iv) Since all the entries in the net
increased further
row of final tal
either negative or zero, the profit cannot be
assembles and sells two
Problem 3-11. Afirmresources. different
typesbe of
desocuritb-beodar
four The production process can
motor, A and B, using
follows : ,
Resources Capacity per month
400 Type Aunits or 250
Motor unit shop resource Type Bunits
any linear combination of the two
175 Type A units
Type A gear and drive shop resource 225 Type B units
Type B gear and drive shop resource
Final assembly resource 200 Type Aunits or 350 Type B
any linear combination of the tuo units or
Type Aunits bring in a profit of Rs. 90 each and type B units, Rs. 60 each.
What should be the optimum product mix ? Formulate the problem, as LP
soluve by simplex method. model and
Solution. The appropriate mathematical formulation of the given problem is:
Maximize 2 = 90x1 + 60x2
subject to the constraints
1 1
400 X1 +950X2 S 1 or 5x1 + 8x, S 2000
s175
X S 225
1 1
200 X1350 X2 S 1l or 7%1 + 4x, S 1400
XË 20, X2 20
where X1 = Number of units of type A
X = Number of units of type B.
Introducing non-negative slack variables s1 20, ST 20 and s3 20 so as to convet
the constraint inequations into equations, the given linear programming problem in
its standard form is :
Maximize Z = 90x] + 60x2 + Os1 + 0s2 + Os3 + Os4
subject to the constraints
5x1 + 8xy + S1 = 2000
+ S = 175
X2 + S3 = 225
7%1 t 4x2 + S4 = 1400
X1, X2, S1, S2, S3, S4 2 0.
An initial basic feasible solution is:
X1 = X2 =0, S1 = 2000, S2 = 175, $ = 225 and sa = 1400.
Starting with this solution, the various Components involved in the algorthn
are shown below :
UNEAR PROGRAMMING 3-31

SIMPLEX TABLEAU I, II, III & IV


90 Minium
Basic Solution 60
ratlo
Variables values S S S SA

S 2000 5 2000/5
175 1 175/1
Sa 225 1 1

S 1400 4 1 1400/7
X, enters
leaves g-Z 90 ‘ 60
& S)
1125 -5
1125/8
8
90
175 1
1 225
225
S3 1 175/4
175 -7

enters Z, 15750 90 90

& SA leaves 60 T - 90

1 9 -2 775/9
775
175
175 1 1 725/4
90
7/4 1 -1/4 7/4
725/4
1 -7/4 1/4
60 175/4
- 15 15
enters 18375 90
S 15 ‘ - 15
&s, leaves G-Z
1/9 1 2/9
S 775/9 2/9
800/9 1 0 -1/9
90 5/36
275/9 0 -7/36
S3 -5/36
1750/9 1 7/36
60 5/3 35/3
19666-66 90
-35/3
-5/3
G-Z
optimum
entries in the C- Z row are either negative or zero, the
Since all the
solution to the given problem has been obtained and is given by :
X1 =
800
9 X2
with maximum Z= Rs. 000 3

aduertising agency wishes to reach two types of audiences.


Problem 3-12. An greater than Rs. 15,000(target audience
A) and
Customers with monthly incomes B). The
with monthly incomes of less than Rs. 15,000 (target audience
advertising costs
customers 2,00,000. One programme of20,000. TV
total aduerntising budget is Rs. For contract
50,000, one programme of radio advertising costs Rs. the number of radio
Ks. outght to be on TV and
reasons, at least 3 programmes5. Survey indicates that asingle TV programme
programmes must be limited to audience A and 50,000 in target
audience B.
reaches 4,50,000 customers in target 80,000 in target
orogramme reaches 20,000 in target audience A and
One radio maximize the total reach.
audience B. Determine the media mix to formulation ofthe given problem is :
Solution. The appropriate mathematical + (20,000 + 80,000) xT
Maximize Z = (4,50,000 + 50,000) x1
constraints
subject to the 2,00,000 5x1 + 2x, S 20
50,000 x) + 20,000 x2 S
Or

23 Or -X1 S- 3
X1
X2.S5
X1, X220
3-32 OPERATIONS RESEARCH: PROBLEMS &
where
X1 = number of programmes on TV
SOLUTIONS
X2 = number of programmes on radio.
Introducing slack variables s 0, S >0 and s3 0, the iterative simplex tal
are as follows
SIMPLEX TABLEAU 1, II, III
5,00,000 1,00,000
G’ Basic Solution
variables values
s S Minimum
ratlo
S 8?20 5 2 0
20/2
Sn -1* 3/1
S 5
x, enters &
Z,
S, leaves
G-Z 5,00,000 ‘ 1,00,000
2 5 5/5
5,00,000 3 -1
S 1 5/1
S, enters &
Z, 15,00,000 5,00,000 -5,00,000
s, leaves C- Z -1,00,000 5,00,000 ‘ 0
2/5 1/5 1
5,00,000 4 2/5 1/5 1
S 5

20,00,000 5,00,000 2,00,000 1,00,000 5,00,000


G-Z -1,00,000 -1,00,000 -5,00,000
The optimum solution is to give 4 programmes on TV and no
radio in order to achieve a maximum reach of 20,00,000 audience. programme on
Problem 3-13. Good Furnishings, afurniture manufacturer produces and sells
desks, chairs and bookshelves. They have no
However, limited availability of machine time, difficulty
labour
in selling their items.
and floor space restrict
production. Data on usage of resources, supplies and profits on items are given
below:
Desk Chairs
Machine hrs/unit 4
Book shelf Supply
5 1000 hrs
Lubcur hrsfunit 3 3 650 hrs
Floor space sq ftunit 9 6
Contribution Rs./unit 270
1260 sq ft
144 225
() Formulate the above as a linear
programming problem.
(ii) Using x), X2, X3 as the respective decision variables,
initiated for the above problem. At one stage, the following tablesimplex algorithm was
Solution 270
was reached.
Basic 144 225
Uariables values
B b=Xg S S S
270 125 1/2
X 5/8 1/8
25
S 1/2 -l/8 - 5/8 1
J35 3/2 27/8 -9/8
9
G-3 225/4 - 135/4

Erom the above table, what ls the current solution ? Is it optimum 2


INEAR PROGRAMMING
3.33

i) If the current solution aboue is not


till an optimum solution is reached. State theoptimum,
optimumcarru out the simplex algorithm
solution.
li) The Personnel Manger of Good Furnishing claims that by
additionallabour force, the profits can be increased. recruiting
Assuming no changes, is the
claimvalid1? Explain with reasons.
solution .)The mathematical formulation of the given problem is :
Maximize Z=270x1 +144x, +225x4
subject to the constraints
8x1 +4x2+ 5x3 S1000
5x1 + 3x2+ 3x3 S 650
9x1 + 6x+ 9x3 S 1260
X1, X2, X3 20,
where x1, X) and xg denote the number of desks, chairs and bookshelf respectively.
(i) From the given tableau, we obtain the current solution as:
X1 : No. of desks =125
S : slack labour hours = 25 hrs.
S3 : slack floor space = 135 sq. ft.
Contribution = 125x 270= Rs. 33,750
Thus solution not optimum as C -Z, has positive quantities in x2, X3 Columns.
(ii) We proceed at follows :
270 144 225
Minimum
Basic Solution ratio
X S S
Cq variables values
5/8 1/8 200
270 125 1 1/2
1/2 -1/8 -5/8 1
0 S 25
27/8* -9/8 1 40
135 0 3/2
9 225/4T -135/4 0
G-Z 0 -5/27
270 100 1 2/9 1/3
8 5/9 -2/3 1 I/27
30
8 4/9 -1/3 8/27
225 40
-16 -15 -16/3
G-Z
optimum
Since all the entries in C- Z, row are either zero or negative, the
solution is arrived at and is given by :
maximum Z=36.000.
X] = 100, x2 = 0, X3 = 40 with
as seen from s = 30 in the
(iv) The solution shows that labour force is slack change
not
basis. Hence, addition of further labour forcenotwillvalid. the optimum solution.
is
The ,personnel Manger's claim, therefore, of three products viz.
KO Solution-3.14. For a companyengaged in the manufacture below :
1, 2 and 3
A, Y, andZthe available dataare given in TablesREQUIREMENTS
TABLE 1. MINIMUM SALES
Product
Minimum sales requirements per unit
10
20
30
OPERATIONS RESEARCH: PROBLEMS &
3-34
REQUIRED PROCESSING TIMES. AND
CAPACm. SOLUTIONS
TABLE 2, OPERATIONS, of Total available
Operations Time (hrs) required per item hours per month
X Y
2 200
1
1 220
2
1 2 180
3
3
(RS.) PER UNIT
TABLE 3. PROFrT
Profit (Rs.) per unit
Product 10
X 15
Y 8

maximize profit.
Find out the product mix to units produced per month for tha
denote the number of
Solution. Let x, yand z appropriate mathematical formulation the
of
the
products X, Y and Z respectively.
problem is :
Maximize Z = 10x + 15y + 8z
subject to the constraints s 200 ...(1)
x + 2y + 22 ...(2)
2x + y + z s 220
...(3)
3x + y + 22 < 180
10 ...(4)
x
20 ...(5)
y ...(6)
z 30
x 20, y 20, z 0
substitutions.
Let us make the following w + 30, where u, u, w 20
X = u t 10, y= r+ 20, z =
function and constraints (1), (2) and
Substituting these values in objective
the problem becomes :
Maximize 2 = 10u + 15U+ 8w + 640
subject to (u + 10) + 2(u + 20) + 2(w + 30) s 200
2(u + 10) + (u + 20) + (w + 30) s 220
3(u + 10) + (u + 20) + 2(w + 30) s 180
u, , w 0
640
or maximize Z = 10u + 15y + 8w +
subject to u+ 2u + 2w < 90
2u + U + w s 150
3u + U+ 2w < 70
u, , w20

Adding slack variables s1, S2 and S3, we get


Maxirmize Z= 10u +15u +8uw +640 + Os + Os2 + Os3
subject to u+ 2u + 2w + S = 90
2u + Wt S2 = 150
3u + U+ 2w + S3 = 70
UNEAR PROGRAMMING 3-35

The initial feasible solution is obtained by setting u = = W= 0.


The solution is : S1 = 90, s) = 150, s3 = 70, Z = 640.
The solution and further improved solutions are presented in the following
tabular form :
SIMPLEX TABLEAU I. II, II
Bastc Solution 10 15 8 0
variables Minimum
values
W S ratio

90 45
S, 1 2 2 1
150 2 1 0 1 150
S
70 3 0 0 1 70
S 1 2
0
10 15 8 0
G-Z
90
15
45 1/2 1 1 1/2
S 105 3/2 0 -1/2 1 70
1 10
S 25 5/2 1 -1/2

G-3 5/21 -7 -15/2


40 4/5 3/5 0 -1/5
15 -3/5 -1/5 -3/5
S 90
10 2/5 -1/5 2/5
10
0 -8 -7 0 -1
C-4
either zero or negative, table II
Since all the elements in the C-2, row are 10,
u V= 40 and w = 0.
gives an optimum solution which is given by:
Subtituting these values, we have
and z=0+30 = 30,
x = 10 + 10= 20, y = 40 + 20 =60,
and the value of objective function is given by :
2 = 10 x20 + 15 x 60 + 8x 30 = Rs. 1340.
units
20 units of X, 60units of Y and 30
The optimum product mix is to produce
of Z to get a profit of Rs. 1340.
Problem 3·15. Minimize Z= 4x1 + x2
subject to the constraints
3x, + 4x 2 20
- Xj - 5x S- 15
X], X, 2 0
as follows :
Solution. The problem may be rewritten
Minimize Z= 4x1 + X2
subject to the constraints
3x, + 4x 2 20
xË +5x, 15
is to convert the inequality
constraint into
first step in the simplex method referred
The
constraint. In minimization problems the slack variables are actuallycapacity,
equality Rather than representing unused
to as surplus variables (negative slack).
which a particularrequirement is met.
they represent the excess amount by denoting urplus variables by s) and so.
Converting inequalities into equalities and
we get
3x + 4x- S1 = 20
15
X1+ 5x2- $2 =
3.36
OPERATIONS RESEARCH:
PROBLEMS &
By doing so,
which violates the
we quickly find that the surplus variables take on
non-negativity restrictions. To overcome this
SOLUTIONS
negative values
to add an additional variable to each constraint that has a
variable. These new variables are called artificial variablesnegative
because prthey
value withweoblem, need
emphasised thsaturplus
convert to origin artificially from infeasible to feasible. It may be are used to
use of an artificial variable is not limited to minimization problems. An
may be used in maximization problem as well. In general as long artificial the
greater than or equal to constraint is involved, regardless of
as at
least one variable
maximisation problems, artificial variable is used. In our problem,
variable, the constraints become:
3x1 + 4x - S1 + Aj = 20
minimizatiarotnificial
introducing or

X1 + 5x2 - S2 +A2 = 15
The simplex method then selects the artificial variables as the
variables. Therefore, the decision and surplus variables are non-basic variablo
can be set equal to zero. Thus, we notice that the addition of the
initial basic
has permitted us to convert the origin from infeasible point to feasible
correct this problem, we must add each artificial variables to the objective
artificial one
variables
To ensure that the artificial variables are not basic variables in funch
we assign themn very high costs. One convenient way ot the optimum solution
each artificial variable a cost of M, where M is defined to be a doing this is to assion
very
should be emphasised that the artificial variables are used only as large number. t
convenience to obtain an initial basic feasible solution. This is the reason a mathematical
name artificial has been given to these variables since they are why the
physical meaning for the original problem. Introducing artificial fictitious and have no
objective function, we get variables in the
Minimize Z= 4x1 + x2 + Os] + Os2 + MA, + MA2
subject to the constraints
3x1 + 4x2 - S1 + Aj = 20
X1 + 5x2 S2 + A2 = 15
X1, X2, S1, S2, A1, A2 >0
We obtain the initial basic feasible
solution by setting :
X1 = X2 = S1 = S2 = 0so that A1 =
Initial feasible solution is summarized in simplex 20 and A, =15.
TABLEI: INITLAL tableau below :
I
SIMPLEX TABLEAU
4
M M Minimum
Basic
variables
Solution
values s, ratio
S A, Ag
M Aj 20 3 4 -1 1 20/4
M
Ag 15
5 -1
15/5 1
Key row
4M 9M -M -M
M M
(Index row) C -2 4-4M 1-9M M M
*Key number

Key column
As is clear from the table, x2 will enter the basis as its corresponding entryin
C-Z, row is most negative and A2 will depart from the solution as it correspondsto
the mínimum positive ratio. For
this applying row
Ro (new)’ R2 (old)/5, RË operations :
(new) ’ R, (old) R2 (new)
UNEAR PROGRAMMING-Jl
3-37
Now we construct the next
simplex table Il which is shown below
TABLE II:SECOND SIMPLEX
4
TABLEAU 1
M M Minimum
Basic Solution
variables values ratlo
S A, A,
M Aj -1
5
5/11
Key rolw
1 3 1 3
- 1/5
11M 1 4M 1
-M M
5 5
(ndex row) C -2 9-11M M 1-4M
5
5

Key column
As it is clear from the table, x enters the basis and. A, leaves it. For this
applying row operations :
RË (new) -R (old) /.Rz (new) ’Rz (old)- RË (new)
We again construct the simplex tableau II as follows :
TABLE III:THIRD SIMPLEX TABLEAU
4 M M Minimum
Basic Solution ratio
CB S A,
Variabtes values
40 5
4
11
25
1 Key row
11

13
4 1 11

19 13
G-4 11

Key column
Now s enters the basis and x leaves it. For this applying row operations :
R, (new) ’ R (old)/ and R2 (new) ’ R2 (old) + R1 (new)
We again.construct the simplex tableau IV as follows :
TABLE IV:FINAL SIMPLEX TABLEAU
4 1 M M

CB Basic Solution S S A, A,
variables values

11 1
S 10 4

3
5 4

13 4
G-4 4
3-38 OPERATIONS RESEARCH : PROBLEMS

Since all the entries in the C, -Z, row are either positive or zero, the &SOLUTONS
opt
solution to the problem is obtained which is : x1 = 0, X2 = 5 with minimum 2E
Problem 3-16. Acompany possesses two manufacturing plants, each of thb
imurn
can produce three products X, Yand Zfrom a common raw material.
proportions in which the products are produced are different in each However,
plant andtheso
are the plant's operation costs per hour. Data on production per hour and costs n
given below, together with current orders in hand for each product.
Products Operating cost per hour
X Y Rs.
Plant A 2 4 3 9
Plant B 4 3 2 10
Orders on hand 50 24 60

You are required to use the simplex method to find the number of production
hours needed to fulfil the orders in hand at minimum cost.
Solution. The problem,stated in an appropriate mathematical form based upon
the given data is as follows :
Minimize (total cost) Z = 9x1 + 10x2
subject to the constraints
2x1 + 4x > 50 or XË + 2x2 25,
4x1 + 3x2 2 24, 3x1 + 2x 260; x1 20, x2 >0
where X = Number of operating hours used in plant A;
X = Number of operating hours used in plant B
Introducing surplus and artificial variables, the final cost function and constraint
equations for the first simplex tableau are:
Minimize Z=9x1 + 10x2 + 0 s1 +0 s2+0s3 + MAj + MA2 + MA3
subject to the constraints
X1 + 2x, - S1 + A, = 25
4x1 + 3xz s2 + Ag = 24
3x1 + 2x - S3 + A3 = 60
X1, X2, S1, S2, S3, A1, Az, A3 20
From the above system of equations, a starting basic teasiblesolution is :
Aj = 25, Az = 24 and A3 = 60
which can be displayed in the following simplex tableau :
TABLE1:INITIAL SIMPLEX TABLEAU
Min.
10 M M M
ratio
Baslc Solutlon S S A, A, Ag
Varlables Dalues 25
25 2 -1
M A, 1 6
M 24 3 -1 1 Keyrow
20
3
M Ag 860 2 -1
8M 7M -M -M -M M M M
9-8M 10-7M M M
G-Z
Key column
LINEAR PROGRAMMING 3-39

TABLE II :SECOND SIMPLEX TABLEAU Untroduce x, and replace A,)


9 10 M M Min.
ratio
Basic Solution S S A, A, Ag
variables values

1 76
19 -1
M A, 4

3
6 1 4
4
-1 1 564
42
M Ag
9 27 -M 9 -M M
M+ 4 M
G-Z - M+84 M - M M

S, and drop A)
TABLE III: THIRD SIMPLEX TABLEAU (Introduce
M M M Min.
10
ratio
S3 A, A, A,
C Basic Solution S S
variables Values
5
4 1 t 4/3
A, 5 3 2
M
20
2 4/3
20 1 3
4
1
S 56
M M
9 4M +6 -M
3
4M +4 M
G-Z 3
M +3

TABLEAU (Introduce x, and replace A,)


TABLE IV:FINAL SIMPLEX M
M M
10
S A, A
S S
Bastc Solution
varlables Values
3
10 15 4 4
4
35 2
5
229 4
4
-3 -2
10
3 0 2
G-4
either zero or positive, we have arrived
Since all entries in the C -2, row are all plant B are
solution, i.e., 17-5 hours of plant A and 3-75 hours of
at the optimum minimum cost of Rs. 195.
needed to fulfill the orders on hand at raw
Problem 3-17. Aproduct is manufactured by blending three different
Given the
should meet certain quality requirements.
materials, The finished productreconimendation with regard to quantity for raw
following data what is your
3.40 OPERATIONS RESEARCH:
PROBLEMS &
materials to be blended, which will meet the quality requirements with
SOLUTIO%
cost.

Qualty characterlstics Contrtbution to quallly


minimum
Mintmum quality
A B C requtrements
3 1
2 5 1 2 15
3 1 2

Cost of raw materlals 2 5 3


per untit (in Rs.)

Solution. The appropriate mathematical formulation of the above problem is:


Minimize (cost) Z = 2x1 + 5x + 3x3
subject to the constraints
3x1 X3 2 10
5x1 + X2 + 2x3 15
X] + 2x 28
where X1 20, X2 20, X3 >0
Introducting surplus and artificial variables, the final cost function and constraint
equations for the first simplex tableau are :
Minimize Z - 2x1 + 5x, + 3x3 + 0 sj +0 s2 + 0 s3 +MA, + MA, + MAz
subject to the constraints
3x1 X3 - S1 + Aj = 10
5x1 + X2 + Zx3 - S2 + A = 15
X1 + 2x S3 + A3 = 8
X1, X2, X3, S1, S2, S3, A1, A2, A3 2 0
From the above system of equations, starting basic feasible solution is :
Aj =10 A2 = 15 and A3 = 8
which can be displayed in the following simplex tableau I.
TABLE I: INITIAL SIMPLEX TABLEAU
2 5 3 Min.
M
ratlo
Basic Solutlon Az As
Uarlables values S A,
0
M A, 10 3 1 -1 3
M A, 15 (6) 2 -1 1 37
row
Key
M Ag 2 -1
9M 3M 3M -M -M -M M M M

G-4 2-9M 5-3M 3-3M M M M

Key column
LINEAR,PROGRAMMING
mARILE Il:SECOND 3.41
SIMPLEX TABLEAU (Introducex, and
3 replace A)
0 M
Basic Solution M M
G variables values S Min.
3
A, A, Ay ratio

M Aj
0 1
X 3 1 2
2 5
15
A 5 2
M 5 5 -1 0 1 25

6M 2 3M 4
55 -M 4M 2
5 5 5--M M M Key row
23 GM 11 3M 4M 2
G-4 5 555 M
55 M

Key column
TABLE II:THIRD SIMPLEX TABLEAU
([ntroduce x, and replace A)
2 5 3
M M M
Basic Solution Min.
C
variables values S, S S A, A, As ratio

8
M A, 0 -1 2. 1

22 4
2 1 0 2
9 0 Key
9

5 2 5
1
9 81
M 2
2 5 -M 2M 1 M 23
M
39 3 g 9

0 M 1 2M M 23
M
9 9 3 39
Key column

TABLE IV:FOURTH SIMPLEX TABLEAU (Introduce s, and replace A,)


2 5 3 0 M M M Min.
Bastc Solution ratio
varlables values S S S A, A, A,
0
4
2

2 30
9
1 0

5 77 154
1
9 6

1 5 Key row
5
6

19 5
6

Key column
3-42 OPERATIONS RESEARCH: PROBLEMS &SOLUTONG

and replace x,)


TABLE IV: FINAL SIMPLEX TABLEAU (Tntroduce s,
M M
G 2 5 3 M
Bastc Solutlon S S S A, A, As
Variables values
81 -2 1 -5

184 2 -1
2 1
9

154 6 -1 -3
S 3
4 -2
2
1 3 2
G-Z
sol.
Since all the entries in C, -Z, row are either zero or positive, the optimum
184 and minimum value of Z is 368,
tion is obtained and is given by : X] = , x2 = X3 = 0 9
Problem 3-18. A chemical company produces two conmpounds A and B. The
D per kg. of compounds A and B
following table gives the units of ingredients C andcost per kg of A and B. Using the
as well as minimum requirements of C and DBand a supply of Cand D
simplex method, find the quantities of A and which would give
at minimum cost. Minimum
Compound
requirement
A B
1 2 80
Ingredient C
3 1 75
D
4 6
Cost per kg problem is :
Solution. The appropriate mathematical formulation of the given
Minimize Z = 4x1 + 6x2 +0s1 + 0 s2 + MA, + M A2
subject to the constraints
X1 + 2x2 S1 + 0S2 + A, +0 A, = 80
3x1 + x2 + 0S1 - S2 + 0 Aj + A = 75
X1, X2, S], S2, A1, A 20
Accordingly, we obtain the simplex tables as follows:
SIMPLEX TABLEAU I, II 8 II
4 6 M M Minimum
ratlo
Basic Solution S S Aj A,
Variables values
80 2 -1 80
M A,
75 3 1 -1 25’
M Az
4M 3M -M -M M

C-Z 4-4MT 6-3M M M

M A, 55 -1
3

4 X 25

5M + 4 -M M- 4
4 M
3

C -Z 14-5M,
3
M 4 - M
3
LINEAR PROGRAMMING |
3.43
6 33 3
1
5
4 14 1 2
5
254 4 6 14
5
14 2
5 5

Since all entries in the C -Z, row are either zero or positive, the optimum
altion is obtained and is given by : x1 = 14, x, = 33 and minimum Z = 254.
14 ka of A and 33 kg of Bshould be produced in order to have a Thus
Rs. 254. minimum cost of
Problem 3-19. Asmall jewellery manufacturing company
is a highlyskilled gem cutter, and it wishes to use this personemploys
a person who
at least 6 hours per
dau for this purpose. On the other hand, the polishing facilities can be used in any
amount upto 8 hours per day. The company specializes in three kinds of
semiprecious stones P, Qand R. Relevant cutting, polishing, and cost requirements
are listed in the following table. How many gemstones of each type should be
processed each day to minimize the cost of the finished stones ? What is the
minimum cost?
P R

Cutting 2 hr 1hr 1hr

Polishing 1hr 1hr 2 hr

Cost per stone Rs. 30 Rs. 30 Rs. 10

Solution. Let xË, x, and x3 represent the number of type P, Qand R stones
finished per day, respectively, then the appropriate mathematical formulation of the
given linear programming problem is :
Minimize Z= 30x + 30x2 + 10x3
subject to the constraints
2x1 + x2 + X3 26
X] + X2+ 2x3 S 8
X1 0, X2 >0, X3 20
Introducing slack, surplus and artificial variables, the modified problem is :
Minimize Z = 30xX) + 30x2 + 10x3 +0s1 + MA, + 0
S2

subject to the constraints


=6
2x1 + X2 t X3 -S1 + A,
S = 8
X] + x + 2x3 +
20
X], X2, X3, S1, S2, Aj
basic feasible solution is :
From the above system of equations, astarting
A, =6 and s = 8
3-44
OPERATIONS RESEARCH:
PROBLEMS 8
which can be displaved in the following table
SIMPLEX TABLEAU I, II & II
30 30 10 M
s&SOLUTIONS
C Basic
variables
Solution
values A, Sy Minratloimum
+A, 6
(2) 1 1 -1

8 1 2
x, enters C -Z 30-2M 30-M 10-M M 0
& A, leaves ‘
30 1
X 3 1 0
6
5 3
2
3
X, enters G-Z 0 15 -5 15
& s, leaves
30 4 1
3

10
3
2
50 0
G-Z 50 10
3 3

Since all entries in the C;-2, row are either


solution to the problem is given by : x1 =, x) =0, xg zero
= or positive, the optimum
with minimum 2 =
i.e., a minimum cost of Rs. 73; for gemstones will be
Band 3, type C stones are realized if 1type A, no yP
processed each day.
Problem 3-20. Afirm plans to purchase at least 200 quintals of
high quality metal X and low scrap conta
purchased must contain at leastquality metal Y. It decides that the
100 quintals of X-metal and no scrap 6
quintals of Y-metal. The can purchase the scrap from two
unlimited quantities. Thefirmpercentage
more r i
of Xand Y metals in termssuppliers A andofthe
of weight
scraps supplied by Aand B is given below:
Metals
X Supplier A Supplier B
25% 75%
1%
The price of A's scrap is Rs. 200 per quintal and that of B's20%
400 perquintal Rs.
The firm wants to determine the quantities that it should buy from thetwo Supplers
So that total cost is
minimized. Using the linear solveit by
graphic method. technique Aprit198
-Solution. The programming
Delhi Uniu. M.BA, A
appropriate
Minimize (total cost) mathematical formulation of the
subject to the constraints Z= 200x] +400x2
b problem
X1 + X2 2 200
~x1 +x 2100 or xË
1. + 3x2 2 400
10 X1 +/x, S 35
or x1 + 2x, s 350
where X1 =The X1 20, X> 20
Xo = The amount ofof scrap (in kg)
amount scrap (in kg) purchased from supplier A
purchased from supplier B
LINEAR PROGRAMMINGH 3.45

lotroducing slack, surplus and artificial variables, the modified or reformulated


problem is :
linear programming
Minimize Z= 20Ox1 + 400x2 + 0S1 +0 s +0S3 + MA, + MA2
subject to the constraints
X] + X2- Sj + Aj = 200
X1 + 3x2 - S2 + A, = 400
x1 + Z2 t S3 = 350; X], X2, S1, S2, Aj, A2
>0
Erom the above system of equations, a starting basic feasible solution is :
A1 = 200, A2 = 400, s3 = 350.
The iterative simplex tables are:
SIMPLEX TABLEAU I, II & III
200 400 M M Min.
Basic Solution ratio
C S S3 A, A,
Variables values

M A, 200 -1 l 200
400
M 400 3* -1 1 3

, enters
350
2M 4M -M -M M M
-
175

& A, leaves C, -Z 200-2M 400-4M M M

M A, |
200
3
-1
3
100
400
400 1
400 3 3
250 1 2 250
1
S3 3 3

2M + 400 M- 400
X enters 400 -M M
3
200-2M 400-M
& A, leaves C-Z 3
M

3 1
200 100
1
400 100
1
50 1
2 2

200 400 -100-100


100 100

Since all entries in the C, -Z, row are either zero or positive, the optimum
Ounon is obtained and is given by : xj = 100, x2 = 100 and minimum Z = 6,000.
Problem 3-21. Maximize 2= X1 + 2x2 + 3x,
subject to the constraints
X1 - X2 t X324, x; + x + 2x3 s8, x1 - X3 22; X], X2, X3 20
Solution. Introducingslack, surplus and artificial variables, the problem becomes :
Maximize Z x1 + 2x> + 3x3 + 0 S) +0s2 + 0 s3 - MA] MA2 - MA3
subject to the constraints
X]- X2 + X3 - S1 + Aj = 4
X1 + x2 + 2x3 + S = 8
X1- X3 - S3 + Ag = 2;x], X2, X3, S1l, S2, A1, A2 2 0
OPERATIONS RESEARCH: & PROBLEMS
3.46
From the above system, a starting basic feasible solution is : Aj = 4, So = 8, Ag -2 SOLUTIONS
which can be displaved in the following table.
TABLEAU I, II, Ii!
SIMPLEX -M -M

Gt
Basic Solution
variables values
4
X
-1 1
S,
-1
Sy

1
A,
1
A,
MInrattoimum
4
-M A, 2
S 1 -1 1
2 1 -M 2+
-M +A, 1+2M 2- M 3 -M
A, enters &
A, leaves C- Z -1
1
le
-1 2
M -A, 1
1 2
6
S 0 -e
2 -1
-M 1+ M
2 -M 2+ 2M
x enters &
A, leaves C- Z
1
3
1 0 1 2
2 6
5 1
3
S 2
1
3
4 2
x, enters &
S, leaves G-Z
3
6
2 5
18 1 5

C-Z -
the optimum
Since all entries in the C; -Z, row are either zero or negative,
solution is obtained and is given by :
18 6 54
X} = X2 = ~ X3 = and minimum Z=
Problem3-22. A bicycle manufacturer makes two models--a sports cycle anu
are needed, while a racis
racer. In order to make asports model, 6 man hours more than 15 men
model requires 10 man hours. The manufacturer can employ no of material
and these men work 8 hours per day for 6 days each week. The cost such materials
amounts to Rs. 500 per cycle and the manufacturer's weekly quota of 'sports
exceed Rs. 40,000. The firm has a at least 30made in
may not contract to supply should be
models' and 20 'racers' per week. How many cycles of each type cycleis Rs
order to obtain the maximum possible profit, if the profit on each sports
100 and on each racing model is Rs. 300 problemisas
Solution. The appropriate mathematical formulation of the given
follows :
Maximize (total profit) Z = 100x1 + 300x2
subject to the constraints
6x) + 10x, s 720
500x) + 500x2 S 40,000
2 30
x22 20 ;x0, xz >0
UNEAR PROGRAMMING -| 3.47

where x1 = Number of Sports cycles to be manufactured.


x) = Number of Racing cycles to be manufactured.
Introducing slack, surplus and artificial variables, the final profit function and
constraint equations for the first tableau are :
Maximize Z-100x1 +300x2 +0s1 +0 $2 +0 sg - MAj +0 s4 - MA2
subject to the constraints
6x1 + 10x + S1 = 720
50x1 + 50x, + S2 = 4000
X1 - S3 + Aj1 = 30
-S4 + A = 20
X], X2, S1, S2, S3, S4, Aj, A2, Ag20
Astarting basic feasible solution is : S = 720, $, = 4000, A, =30 and Ag = 20.
SIMPLEX TABLEAU I, I, II& V
Basic Solution 100 300 -M 0 -M Minimum
G ratio
variables values S S S3 A S4 A,
B
10 1 0 0 720/10
S 720 6
4000 50 50
4000/50
S 20/1
30 -1
-M A 1
-1
20 1* 0
-M
- M - M 0 - M - M
|, enters & Z,
|Ag leaves G-Z 0 - M -M
100+M 300+M

6 0 520/6
520
0 0 50 3000/50
S 3000 50
30 1 0 -l 30/14
- M
300 20 1 -1

300 0 M - M -300
X, enters & Z, -M
100 + M - M 300
A, leaves G-Z
0 1 0 6 340/10
S 340
0 50 50* 1500/50
-S, 1500
30 1 0 -1
100 -Ve
300 20 -1
0
300 0 0 -100 300
S, enters & Z, 900 100
0 100 300
S, leaves C-Z
1 0
40 0 -4
S
0 S 830 0 50
100 830 0 0 5 -1
1
300 50 1
0 0
8 50
6 200 0
(ndex 3
1800 100
0
300
-6 -200 0
row)
optimum
Since all entries in the C-Z, row are either negative or zero, anHence, the
solution has been obtained and is given by : xË = 30 and x2 = 50.
racing
optimum solution is to produce 30 units of sports cycles and 50 units of
cycles to get a maximum profit of Rs. 1800.
Problem 3.23. A firm has an aduertising budget of Rs. 7,20, 000. It wishes to
allocate this budget to two media : magazines and television, so that total exposure
3.48 OPERATIONS RESEARCH:
:PROBLEMS &SOLUTIONS
is maximized. Each page of magazine advertising is estimated to
resultin in
exposures, whereas each spot on television is estimated to result 60,000
exposures. Each page of magazine advertising costs Rs. 9,000 and each
television costs Rs. 12,000. An additional condition that the
that at least two pages of magazine advertising be used and firm has spot ons
1,20,000
at least
television. Determine the optimum media-mix for this firm. (Delhi Uniu. M. 3B. spots on
specified
A, 199)
Solution. Maximize (total expoSure) Z = 60,000 x1 + 120,000 x,
subject to the constraints
9,000x + 12,000x, s 720,000 or 3x1 + 4x, s 240
X1 22
X2 2 3
X1, X2 20
where
x1= Number of pages of magazine
x)= Number of spots on television
Introducing slack, surplus and artificial variables, the problem can be expressed in
standard form as:
Maximize Z = 60,000x1 + 120,000x2 + 0.Sj + 0.s2 +0.s3- M. A1 - M. A,
subject to the constraints
3x1 + 4x + S1 = 240, xË - S2 + A1 = 2, x2 - S3 + A2 = 3
X1, X2, S1, S2, S3, A1, A2 20.
From the above system of equations, a starting basic feasible
S] = 240, A, = 2 and A, =3. solution is :
SIMPLEX TABLEAU I II, III & IV
Basic Solution 60,000 120,000 -M - M Minimum
Dartables Dalues S ratio
Sn Sg A, Ag
S 240 3 4 1 0 60
- M A 2
-] 1
-M 3
34
-1
enters M
- M M
& A, lcaves C- Z 60,000+M 120,000+MÍ 0 M - M - M
- M -M
S 228 3 0
4 76
- M 2
1°) -1 2
120,000 3 1
-1
X, cnters -M
60.000 + M
120,000 M - 120,000 -M
& A, leaves C- Z
- M
120,000
S 222
3 555
60,000 2
120.000 3 -1

S, enters 60,000 -1
120,000 -60,000 -120,.000
& s,leaves G
S3 55-5 60,000 120,000T
2 1 3
60.000
58-5 4 4
120,000

3
4
60,000
120,000 30,000 30,000
G-4 30,000 30,000
LINEAR PROGRAMMING
349
Since all entries in the C-2 row are either zero or
solution is x1 = 2, X) = 585 and max. 2 -7,14,000. negative, an optimum
Problem 3-24. A carpenter has started a
bandcarts. Each cart consists of a frame and twoworkshop
wheels.
in which he manufactures
A frame uses 3 hours and
uheel 2 hours of labour of which 90 hours per week are
uuonts that he should manufacture at least 10 carts during available. The carpenter
a week. The cost of the
frame is Rs. 500 ant that of a wheel is Rs. 200. Formulate this as a linear
programming model and using Simplex Method, determine an optimum production
plan.
Solution. The appropriate mathematical formulation of the given problem is:
Minimize Z= 500x1 + 200x2
subject to the constraints
3x1 + 2xy S 90
XË 2 10
X2 = Zx1; x1, X) 2 0
Introducing slack, surplus and artificial variables, the problem can be expressed in
standard form as:
MinimizeZ= 500x1 + 200x2 +0.s1 + 0.s2 + MA, + MAz
subject to the constraints
3x1 + 2x2 + $1 = 90
X1 - S2 +Aj = 10
-2x1 + X2 + Az= 0; x1, X2, S1, Sz, A1, Ag 2 0
From the above system of equations, a starting basic feasible solution is :
Sj = 90, Aj = 10 and A, = 0
SIMPLEX TABLEAU I, II, II&IV
Basic Solutlon 500 200 M M Minimum
A, A, ratio
S
variables values
90 2 0 45
M 10 0 -1 1
A,
M -2 0

-M M 0 -M M M
, enters & Z, M
500+M 200 -M
|A, leaves G-Z
90 7 90
7
M +A, 10 (1) -1
10 -
200 0 -2

M-400 200 -M M
enters & Z, 900-M 0 M
|A, leaves G-Z
20 0 ) 7
S
-I
500 10
0 -2
200 20
500 200 0 -900
9,000 900
0
G-Z
Since all entries in the C -Z, row are either zero or positive, the optimum
2 = 9000.
SOlution is obtained and is given by : X| = 10, x2 = 20 with minimum
Hence the manufacturer should manufacture 10 frames and 20 wheels at a minimum
Cost of Rs. 90r0.
OPERATIONS RESEAROCH
4:PROBLEMS
3-50
Problem 3-25. XYZ Company during the festival season combines two items A
and Bto form gift packs. Each pack must weigh 5 kg. and should Contain at least 2
&SOLUTONS
kg of A and not more than 4 kg. ofB. The net contribution to the company is Rs.
10 per kg. of A and Rs. 12 per kg. of B. The company wants to determine the
optimum mix.
Formulate the above as a linear programming problem to maximize net
method.
contribution per pack and solve the same by using simplex
Solution. Let x; k. of A and x> kg. of B be mixed per pack. Then the
LP model is :
mathematical formulation of the given problem as
Maximize (total contribution) Z = 10x; + 12x2
subject to the constraints X2 20
X1 + X2 =5, x) > 2, x2 S 4, x1,
variable s, to (31
Subtracting surplus variable s1 from (2), adding slack above LP probi
The
artificial variables Aj and A, to (1) and (2) respectively.
standard form may be rewritten as under:
Maximize Z = 10x1 + 12x +0.S1 + 0.s2 - MA] - MA2
subject to the constraints
X1 + X2 + 0.s1 + 0.s2 + Aj + 0.A= 5
X1 + 0.x2 - S1 + 0.s2 + 0.A, + A, = 2
0.x1 + X, + 0.S1 + S2 + 0.A, + 0.A, = 4
X1, X2, S1, S2, A1, A2 2 0
The initial basic feasible solution is obtained by setting :
s = 4.
X1 = X2 = S1 = S2 = 0 so that A1 =5, A, = 2and
SIMPLEX TABLEAU I, II, III
10 12 -M -M Minimum
Basic Solution ratio
S S A, A,
variables values
5
5 1 1
-M A, 2t
2
-M Az 4
-1
1
undefined
S 1
X enters & 2M -M M -M -M
A, leaves C-Z 10+2M 12+M -M

3 34
-M A1 undefined
10 X1 2 -1
4
S2 4 1
X enters & 10 -M -M-10 -M
A, leaves
C,-24 M+12 M+10

12 3 1
10 2
-1
S2 0 -1
56 10 12 2
C-Z
optimum
Since all entries in the C,-Z row are the
solution is arrived at and is given by: either zero or negative,
X1 = 10, x2 = 12 and maximum Z = 56.
Q
Problem 3-26. An Air Force is experimenting with three types of bombs P.the
and Rin which three kinds of explosives viz., A, B used. Taking o
various factors into and Cwill be most 600 kg.
consideration, it has been decided to use at m
UNEAR PROGRAMMING
3.51
explosive A, at least
480 kg. of explosive Band
Prequires 3, 2, 2 kg. of A, B and Crespectively,exactly
bomb540 kg. of 1,
Qrequires 4, 3 kg. C. ofBomb
explosive A, B
and C respectively and bomb Rrequires 6, 2, 3 kg. of A, B
bomb P will give the equivalent of a 2-ton explosion, bomband Crespectively. Now
explosion and bomb R will givea 4-ton Q will give a 3-ton
(an the Air Force make the biggest bang ? explosion. Under what production schedule
solution. Let X], X2 and x3 be the number of bombs of type P, O and R to be
produced. Then the appropriate mathematical formulation of the given problem is :
Maximize Z = 2x1 + 3x2 + 4x3
subject to the constraints
3x1 + X2 + 6x3 S 600
2x1 + 4x, + 2x32 480
2x1 + 3x2 + 3x3 = 540; X1, X2, X3 >0
Introducing slack, surplus and artificial variables, the problem can be expressed in
standard form as :
Maximize Z = 2x1 + 3x2 + 4x3 + 0sj +0 s -MA, - MA
subject to the constraints
3x1 + X2 + 6x3 + S1 = 600
2x1 + 4x2 + 2x3 -S2 + Aj = 480
2x1 + 3x, + 3x3 + Ag = 540
X1, X2, X3, S1, S2, S3, A1, A2 >0
The initial basic feasible solution is : S = 600, A, = 480 and A, = 540
SIMPLEX TABLEAU I, II, III
Basic Solution 2 3 4 -M -M Minimnum
C’ ratio
variables values A, A,
600 6 1 600
S
-M t-A, 480 2 4 -1 1 120 ¬
-M 540 180
Ag 2 3 3 1

X enters & Z, -4M -7M -5M M -M -M


2+4M 3+7M 4+5M -M
A, leaves C-Z

480 960/1l+

3 120 1 - 240

3 3
1 120
-M 180 4
Ag 2 2

xg enters & Z
3 M
3
_3M
2
-3M
4
-M
2 2
3M
s, leaves G-Z 4

1
860 5 1 1920
4 11 22
11 11
3
840 3 -Ve
3 11
11 11
540 2 3 15. 72
-M 22
11
OPERATIONS RESEARCH: PROBLEMS &
3-52

5 3M 15M
SOLUTIONS
29 2M -M
S, enters & Z, 1111 3 4 1111 1 22
5 3M 7 15M
7 2M 0 11 22
A, leaves G-Z

4 84 27 0 1 5
X 55
1
1
3 Xy 96 5
1 5
2
4 1
0 S 72 15 5
141 4
3
55

31
G-4 55

Since all entries in the C; -2, row are either zero or negative, the optimum
Z = 624.
solution is given by: X1 = 0, X, =96, Xg = 84 with maximum
special mixture
Problem 3-27. XYZ Chemicals Ltd. must produce 10,000 kg. ofa costs
X3. Xj Rs. 8 per
for acustomer. The mix consists of ingredients X1, Xg, and more than 3,000 ka.
kg. and X, cost Rs. 10 per kg. and X3 costs Rs. 11 per kg. No 2,000 k.
of Xcan be used and at least 1,500 kg. of X must be used. Also, at least
of Xg is required.
(i) Calculate the number of kg. for each ingredient to use in order to minimize
total cost for 10,000 kg, and
(i)Calculate the lowest possible total cost.
Solution. The appropriate mathematical formulation of thegiven problem is:
Minimize (total cost) Z= &x1 + 10xy + 1lx;
subject to the constraints
X1 + X2 + X3 = 10,000
X1 s 3,000
2 1,500
X32 2,000
Putting xË =X- X1, xy =Xy + 1500, x3 =X3 + 2,000; the problem becomes :
Minimize 2 = 8X- 8X; + 10X, + 11X3 + const.
subject to the constraints
X-X1 + X+X3 = 6500
X- X; s3000; X, X,,X2, X3 20.
Treating X3 as a slack variable and introducing a slack variable second
S2 in the
constraint, the problem becomes:
Minimize Z= 8X- 8X + 10X, + 11X3 +const.
subject to the constraints
X- X1 +X +X3 = 6500
X-X; +s2 = 3000; X, X, X2, X3, S2 >0.
INEAR PROGRAMMING|
3.53
SIMPLEX TABLEAUI, II & III
Baslc Solutton 10 11 Mintmum
variables values X
X, S ratlo
11 6500 -1 1 6500
3000
-1 3000
G-Z -3 ‘ 3 -1
11 3500
3500 4
X 3000 -1
G-Z -1‘
10 X 3500
1
X 3000 1 1 1
G-Z 1 2

Since all entries in the C, - Z, row are either zero or positive, the
solution is obtained and is given by: optimum
Xo = 3500, X = 3000, X1= 0, X3 = 0, i.e., x = 5000, x1 = 3000; x3 = 20O0.
Minimum Z= 8x 3000 + 10>x 5000 + 11 x 2000 = 96,000.

SELF-ASSESSMENT EXERCISES
1. (a) Two materials A and B are required to construct tables and book cases. For one
table 12 units of A and 16 units of B are needed while for a book case, 16 units
of Aand 8units of Bare required. The profit on abook case is Rs. 25 and Rs. 20
on atable. 100 units of material Aand 80 units of B are available. How many
book cases and tables be produced to have maximum profit.
(b) A manufacturing company contemplates to produce two additional products,
called A and B, which can be marketed at prevailing prices in any reasonable
quantities without difficulty. It is known that product A requires 10 and 5 man
hours per unit in the foundry and machine departments respectively and that
product B requires only 6 and 4. However, the profit margin of A is Rs. 30 per
unit as compared with Rs. 20 per unit for B. In the week immediately ahead, it is
estimated that there will 1000and 600 man hours available in the foundry and
the machine departments respectively. How much of A and B should be produced
in order to most profitably ütilize the excess capacities ? Use simplex method
only.
2. () A firm has available 240, 370 and 180 kilos of wood, plastic and steel
respectively. The firm produces two products A and B. Each unit of product A
The
requires í, 3requirement
corresponding and 2'kilosforofeach
wood,unitplastic and3, steel
of B are 4 and 1respectively.
respectively. If A
sells for rupees 4 and B for rupees 6, determine how many units of A and B
should be produced to obtain the maximum gross income. Formulate the problem
mathematically. Solve the problem by simplex method.
(6) The XYZ company manufactures two products Aand B. These are processed on
same machine. Atakes 10 minutes per item and B takes 2 minutes per item on
machine. Machine can run for a maximum of 35 hours in a week. Product A
requires 1 kg and product B, 0.5 kg. of the raw material per item, the supply of
which is 600 kg per week. Not more than 800 items of product B are required
per week. If the product A costs Rs. 5 per item and can be sold for Rs. 10 and
product B costs Rs. 6 per item and can be sold for Rs. 8 per item, determine bU
simplex method how many items of A and B per week be produced in order to
maximize the profit.
3-54 OPERATIONS RESEARCH: PROBLLEMS &
3. Two products Aand Bare processed on 3 machines M1, My and M3.
SOLUTIONS
The
times per unit, machine availability and profit per unit are:
Machine Auailability
processing
Processing time Any unutilized time on Machine M.
(hrs.) (hours)
can be given on rental basis to
M, 2 3 1500 others at an hourly rate of Rs., 1-50
M, 3 2 1500 Solve the problem by Simplex
method to determine the maximum
M, 1 1 1000 profit.
Profit Rs. 10 Rs. 12
(Indira Gandhi Open Uniu M.BA, Dec. 199
4. A factory manufactures three products which are processed through three different
production stages. The time required to manufacture one unit of each of the thre
products and the daily capacity of the stages are given in the following table
(Time per unit minutes) Stage capacity
Stage (in minutes)
Product 1 Product 2 Product 3

1 1 1 430
2 3 2 460
3 1 4 420

Profit per unit Rs. 3 Rs. 2 Rs. 5

(0 Set the data in simplex table.


(i) Find the table for optimum solution.
(i) State from the table-minimum profit, production pattern and surplus capaciy
at any stage.
(iv) What is the meaning of shadowprice ? Where is it shown in this table ?Explat
it in respect of resource of stages having shadow price.
(u) How many units of other resources will be required so as to completely u
Feb, 19)
the surplus resource ? (Osmania Uniu. M.BA,
product
5. ABC manufacturing company can maketwo products P and P2. Each of the
requires time on a cutting machine and a finishing machine. Relevant data are :
Product
P P2
Cutting hrs. (per unit) 2 1
Finishing hrs. (per unit) 3
Profit per unit Rs. 6 Rs. 4
Maximum sales (units per week) 200
numberof
The number of cutting hours available per week is 390 and the ofeach
finishing hours available per week is 810. How much should be produced
product in order to achieve maximum profit for the company? manufacture
6. Asmall furniture manufacturer produces wooden chairs and tables. ThegeneratesRs
wants to determine the product mix that maximizes
profit. Each chairtwo bottleneck
15 of profit, whereas each table generates Rs. 30 in profit. There are 4
requires
operations in the production process, finishing and assembly. Each chairoffinishing
hours of finishing and 2 hours of assembly. Each table requires 5 hours 240hours
and 4 hours of assembly. There is a capacity of 200 hours in finishing and
and28tables
in assembly. Finally, forecast indicates a demand potential of 40 Determine
Management considers these forecasts as upper bounds on chairs (capacits,
) and

the optimum product mix. (ii) What is the shadow price ofproduction.
assembly capacity ? finishing
INEAR PROGRAMMINGI 3.55

7. Acompany manufactures two kinds of machines. each requiring a different


manufacturing technique. The deluxe machine requires 18 hours of labour, 9 hours of
testing, and yields aprofit of Rs. 400. The standard machine requires 3 hours of
labour, 4 hours of testing, and yields a profit of Rs. 200. There are 800 hours of
labour and 600 hours of testing available each month. A marketing forecast has
shown that the monthly demand for the standard machine to be no more than 150.
Management wants to know the number of each model to produce monthly that will
maximize total profit. Formulate and solve this as a linear programming problem.
8. Afactory engaged in the manufacturing of pistons, rings and valves for which the
profits per unit are Rs. 10, Rs. 6 and Rs. 4 respectively wants to decide the most
profitable mix. It takes 1 hour of preparatory work, 10 hours of machining and 2
hours of packing and allied formalities for a piston. Corresponding time requirements
for rings and valves are 1, 4 & 2 and 1, 5 & 6 hours respectively. The total number
of hours available for preparatory work, machining and packing &allied formalities are
100, 600 and 300 respectively. Determine the most profitable hix, assuming that
whatever is produced can be sold.
9. Noah's Boats makes three different kinds of boats. All can be made profitably in this
company, but the company's monthly production is constrained by the limited
amount of labour, wood and screws available each month. The director will choose
the combination of boats that maximizes his revenue in view of the information given
in the following table -
Input Row Boat Canoe Kayak Monthly auailability
|Labour (Hours) 12 1,260 hours
Wood (Board feet) 22 18 16 19,008 board feet
Screws (Kg) 2 4 3 396 kg.
Selling price (in Rs.) 4,000 2,000 5,000

(a) Formulate the above as a linear programming problem.


(b) Solve it by the simplex method. From the optimum table of the solved linear
programming problem, answer the following questions :
(0 How many boats of each type will be produced and what will be the resulting
revenue?
() Which, if any, of the resources are not fully utilised? If so, how much of spare
capacity is left ?
(i) How much wood will be used to make all of the boats given in the optimum
solution ?
10. Sumant' s professor at L.T. has given him three very long lists of problems with the
instruction to submit no more than 100 of them (correctly solved) for credit. The
problems in the first set are worth 5 points each, those in the second set are worth
4 points each, and those in the third set are worth 6 points each. Sumant know
from experience that he requires on the average 3 minutes to solve a 5-point problem,
2minutes to solve a4-point problem, and 4 minutes to solve a 6-point problem.
Because he has other subjects to worry about, he can not afford to devote more
than 3, hours altogether to this mathematics assignment. Moreover, the first two
sets of problems involve numerical calculations and he knows that he cannot stand
more than 2% hours of work on this type of problem. Under these circumstances,
2
how many problems in each of these categories shall he do in order to get maximum
possible credit for his efforts ?
11. A company manufactures 3 products A, B and C which are manufactured in two
departments (0) and (). The company has enough demand for these products.
However, the production capacity is limited. The relevant data are as follows :
3-56 OPERATIONS RESEARCH:PROBLEMSS&SOLUTIONS
Department (I)
Product Department ()
(hours) (hours) Expected
profithunit
3
A 60
2
2,100
50 1,600
1
C 40 1,300
144
Availability 4,000
Find out the optimum product mix and maximum profit. If the company wants to
expand its production capacity, which production department should be given priorty
and why ?
products, say type A and B. Produet D .
12. A company produces two types ofof a lower quality. Profits on the tunS of
superior quality and product A is
products are Rs. 30 and 40 respectively. The data on resources required, availabil
of resources are given below : Requirements
Product B
Capacity auailable
Product A per month
60 120 12,000
Raw materials (ko)
8 5 600
Machining (hours per piece)
3 4 500
Assembly (man hour)
order to
How should the company manufacture the two types of products in
have a maximum overall profit ?
13. A cabinet manufacturer produces wood cabinets for TV crated.sets, sterosystems and
assembled, decorated and Each TV cabinet
radios, each of which must be 1/10 hour to crate and
requires 3 hours to assemble, 5 hours to decorate and
hours to assemble, 8
returns a profit of Rs. 10. Each stereo cabinet requires 10
hours to decorate and 3/5 hour to crate and returns a profit Rs. 25. Each radio
cabinet requires 1 hour to assemble, 1 hour to decorate and 1/10 hour to crate and
maximum of 30000, 40000and
returns a profit of Rs. 3. The manufacturer has the
120 hours available for assembling, decorating and crating respectively.
() Formulate the above problem as a Linear Programming problem.
(ii) Use simplex method to find how many units of each product should be
manufactured to maximize profit.
(i) Does theproblem have multiple solutions? (Delhi Uniu.. M.BA, Nou. 1998)
14. Product A offers a profit of Rs. 25 per unit and Product B vields a profit of Rs. 40 per
unit. To manufacture the products leather, wood and alue are required in the
amount shown below :
RESOURCES REQUIRED FOR ONE UNIT
Product Leather Woods Glue
(in litres)
(in kg.) (in sq. metres)
0.2
A 0.50 4
B 0.2
.25 7
of wood
Available resources include 2,200 kgs. of leather; 28,000 square metres
and 1,400 litres of glue :
(0 State the objective function and constraints in mathematical form.
(io Find the optimum solution. remains
(iû Which resources are fully consumed ? How much of each resource
unused ?
June1991)
(iv) What are the shadow prices of resources ? (CS FAnal. planning
15. For the annual handmade furniture show, the school of studies is I IYear
vocational
to make furniture for the sale. There are three
woodworking classes - I Year,
chairs-
and II Year, at the school and they have decided to styles of
make three
INEAR PROGRAMMING-| 3-57

Band C. Each chair must receive work in each class and the time in hours required
for each chair in each class is :
Chair I Year II Year IIIYear
A 2 4 3
B 3 3 2
2 1 4

During the next month there will be 120 hours available in the Ist year class,
160 in the Il year class, and 100 hours in ll year class to produce the chairs. The
teacher of the wood-working classes feels that a maximum of 40 chairs can be sold at
the show. The teacher has determined that the profit from each type of chair will be :
A, Rs. 40; B, Rs. 35 and C, Rs. 30.
(a) Formulate a linear programming model to determine how many chairs of each
tupe should be made in order to maximize profits at the show and sale.
(b) Solve the above problem using the simplex method.
precious metals platinum and
16. A company manufactures 3 types of parts which usethe government regulates the
gold. Due to shortage of these precious metals,
to supply,
amount that may be used per day. The relevant data with respect
requirements and profits are summarized in the table below :
Platinum Gold
required/unit requiredunit Profitunit
Product (Rs.)
(gm.) (gm.)
3 500
A 600
4 2
B
6 4 1,200
C
160 gm. and 120 gm. respectively.
Daily allotments of platinum and gold are scarce
of precious metals ? VWhat is the
Howshould the company divide the supply
optimum profit ? and Z. Only
manufacturing company produces three brands of cycles ; X, YDaily supplies
17. A cycle short supply to limit the production.
three type of raw materials are in400 kg., 200 kg. and 300 kg. respectively, whereas
of raw materials A, B and C are for
material A must be used the samne day
material Band C may remain unused,
material required for making each cycle
certain reasons.
per cycle and the amount of
The profit
are given below: Material used (Kg.)
Brand
Profit per unit (Rs.) B
A
2 1 1
140 1
8
300 4 1
2
400

optimum product mix and the profit earned. Can there be any
Find out the daily be equally profitable ? If so, find that
too.
other product mix which may requiring a different
company manufactures two kinds of machines,18each hours of labour,9 hours of
18. A deluxe machine requires hours of
manufacturing technique. The 400. The standard machine requires 3
of Rs.
testing, and yields a profitt and yields a profit of Rs. 200. There are 800 hours of
labour, 4 hours of testing, month. A marketing forecast 150. has
labour and 600 hours of testing available each machine to be no more than
shown that the monthly demand number for the standard produced monthly that
of each model to be
Management wants to know the problem
and solve this as a linear programming
will maximize total profit. Formulate
by simplex method. each type is
manufacturing concern produces two types of garments and each tupe for its
19. Agarment factory. Man-minutes required by
processed in three sections of the
3-58 OPERATIONS RESEARCH: PROBLEMS

processing in different sections, profit yielded by each type and man-minutes availa
SOLUTIONS
in the sections are given in the table below :
Processing man-minutes in
Profit
different sections (Rs. per piece)
Sec 2 Sec 3
Sec 1
15 30 2 10
Garment type I
60 10 2 20
Garment type Il
Available man
1080 630 48
minutes/day
Determine (i) number of daily production of each type so that profit becomes
f
(0) maximum, (i) amount of this maximum profit and (ili) the sections, resources
which are fully utilised.
20. A company is engaged in producing three products vlz., A, B and C. The followine
data are available :
A B C
Products
10 12 15
Net sale price (Rs /unit)
Cost (Rsunit) 10
The wholesaler who is responsible for selling to the customers is to be paid Rs
150 per day irrespective of the quantities sold in each of the products.
The products are processed in three different operations. The time (hrs) required
to produce one product in each of the operations and the daily capacity (hrs) available
for each operation centre are given below :
Operations Products Daily capacity
A B C (hrs)
1 2 3 2 400
2 3 2 2 350
3 1 4 2 300

What product mix would yield maximum profit and how much ?
21. A farmer has 150 acres of land suitable for cultivating crops A and B. The cost of
cultivating crop Ais Rs. 400per acre, whereas that of crop B is Rs. 600 per acre.
The farmer has a maximum of Rs. 74000 available for land cultivation. Each acre of
crop A requires 20 hours of labour and each acre of crop B requires 25 hours of
labour. The farmer has a maximum of 3300 hours of labour available. He has also
decided that he will cultivate at least 70 acres of crop A. If he expects to make a profit
of Rs. 150 per acre on crop A and Rs. 200 per acre on crop B, how many acres of
each crop should he plant in order to maximize the profit ?
22. A factory produces three different products viz. A, B, and C, the profit (Rs.) per unit
of which are 3, 4, and 6 respectively. The products are processed in three operations
viz. X, Y and Z and the time (hour) required in each operation for each unit is given
below :
Operations Products
A B C

4 6
Y 5 3
1 2

The product works 25 days in a month, at the rate of 16 hours a day in two
shifts. The effective working of all the processes is only 80% due to assignaole
causes like power cut and breakdown of machines. The factory has 3 machines n
operation X, 2machines in operation Y and one machine in operation Z. Find out the
optimum product mix for the month.
UNEAR PROGRAMMING 3-59

14. Consider the following linear programming problem :


Maximize Z 3x1 + 2x + 5x3
subject to the constraints
x, +2x, +X, S a,
3x +2x,S a,
x, + 4x, S a,
where aj, ag and ag are constants. For specific values of aj,ap and ag, the optimum
solution is :
Basic Solution 3 2 5
variables values X3 S1
Cgl
2 X2 100 bË 1 1 1
2
0 1 1
5 X3 b2
2
0 -2 1 1
20 b3
1350 |4 0 C1 C2

where b1, b2, b3 and cj, C2, C3 are constants.


Determine :
(a) Values of aj, az, and az that yields the given optimum solution.
(b) Values of bË, b2,b3 and c1, c2, C3 in the optimum table.
24. The simplex tableau for a maximization problem of linear programming is given below :
C Product mix X1 X2 S1 S2 Quantity
5 1 1 1 10
X2
1 3

C 4 5 0
5 5 5 50
5
G-Z -1

Answer the following questions, giving reasons in brief :


() Is the above solution optimum ?
(i1) Are there more than one optimum solution ?
(u) Is this solution degenerate ?
(u) Is this solution feasible ?
slack in machine A (in hours/week) and so is slack in machine B (in
(u) If s is which
hours/week), of these machines is being used to the full capacity when
producing according to this solution ?
(ui) A customer would like to have one unit of product x and is willing to pay in
increased
excess of the normal price in order to get it. How much should the price be
inorder to ensure no reduction of profit ?
(uli) How many units of the two products? xË and x2 are being produced according
to this solution and what is the total profit
(vili) Machine A (associated with slack S1 in, hours/week) has to be shut down
for repairs for 2 hours next week. What will be the effect on profits ?
(x) How much would you be prepared to pay for another hour (per week) of
capacity each on machine A and machine B ?
25. Given below are the objective function, the constraints and the final simplex tableau
tor a linear programming product-mix problem :
Objective function :
Maximize Z2x1+ 5x2 + 8x3 +0.s1 +0.s2 +0.s3
OPERATIONS RESEARCH: PROBLEMSs &
3-60
SOLUTIONS
subject to the constraints (hours, Department I)
6x, + &x, + 4x,s 96
(hours, Department I)
2x, + x, + 2x,s 40
(hours, Department lI)
5x, + 3x, + 2r,s 60
x3 20.
X], x2,FINAL SIMPLEX TABLEAU

2 5
G s2 $3
CB Product x2 x3 S1
Quantity
mix
1/6 -1/3
1/3 8/3
-1/12 2/3
5/6 1
-1/3 -1/3
56/3
Sg 7/3 44/3
8 1/6 11/3
25/3
-19/3
5
-1/6 -11/3
1625
C,-Z
contribution shown bu
() Write the optimum product mix and the profit
above solution.
reason(s).
(ii) Is this solution feasible ? Why ? Give brief show one
(iti) Does the problem have any alternative optimum solution ? If so,
such solution.
(iv) Indicate the shadow prices of three departments.
(u) If the company wishes to expand the production capacity, which of the
three departments should be given priority.
(vi) If the company produces 6 units of x1, how many units of x2 and x3 shall
have to be reduced, if any ?
(uii) If a customer is prepared to pay higher prices for product x1,how much
should the price be increased so that the company's profit remains unchanged ?
(viil) What are the marketing implications ?
(ix) If s1 is slack in department I and s is slack in department Il and s is slack in
department II, which of these departments is being used to the full capacity when
producing according to this solution ?
26. A firm is planning to produce two products A and B which contributes Rs. 15 and Rs.
20 per unit respectively. Each product has to pass through 3 departments and the
related information about them is given below :
Hours required Maximum auailalble
Departments Product A Product B hours
1 2 1500 Hours
2 3 2 1500 Hours
3 1 1-5 600 Hours

Find the optimum solution and answer the following :


(0 Is the solution obtained feasible ?
(i0 ls the solution obtained degenerate solution ?
(i) What are used and unused departments hours ?
(iv) What are the limiting factors and their associated opportunity cost
the
(u) Due to some reason, two hours of department 2 are lost. Should
management revise its product--mix ? If yes, suggest the new one. is
(u) The Marketing department argues that since Product B's contribution
argumentis
more, theproduction of B should also be more than that of A. If this
accepted, how much the firm will suffer, if at all there is a loss ?
INEAR PROGRAMMING--| 3-61

Formulate linear programming model for the following problen and solve the problem
27. using simplex method.
The
A company sells two types of fertilizers, one liquid and the other isjardry.
and the
liguid fertilizer contains 2 units of chemical Aand 4units of chemical Bper
dry fertilizer contains 3 units of each of the chemicals Aand Bper carton. The igu
fertilizer sells tor Rs. 3per jar and the dry fertilizer sells for Rs. 4 per carton. A tarner
chemical B
requires at least 90 units of the chemicals Aand at least 102 units of the
for his farm. How many of the each type of fertilizers should the farmer purchase to
min1mize the cost while meeting his requirements ? at least
Ahealth store desires to produce acapsule of natural vitamins that contains
12 units of vitamin A and no less than 16 units of vitamin B. Two ingredients are
available in ample supply to produce the specified vitamin capsule. Each ingredient
contains both vitamins A and B, and the capsule can be produced using any one
ingredient or a combination of the two. Each am. of the ingredient one contains 3
am, of the second ingrediernt on
units of vitamin A, and twO units of vitamin B. One
vitamin B. If the first
the other hand, contains 2~unit of vitamin Aand 1 unit of
4 per gm., what
ingredient costs Rs. 6 per gm. and the second ingredient costs Rs.
is the minimum cost of producing the capsule ?
patients. The dietician wishes to
29. A nursing home dietician is planning menus for fat, 60 units of carbohydrates, 90
provide a diet which has a minimum of 120 units ofcholesterol per patient per month.
units of proteins and a maximum of 70 units of Type 1contains 3 units of fat, 2
These goals can be met using two types of food.
units of carbohydrates, 6 units of proteins and 2 unitsunits of cholesterol per kg., while
each kg. of type l contains 12 units of fat, 4 of carbohydrates, 3 units of
proteins and 3 units of cholesterol. The cost of type per I food is Rs. 3 per kg. and of
type II food Rs. 3-75 per kg. How many kilograms month of each type of food
should be used to minimize the cost?
and R. Each
arrange a special diet using three foods P, Qvitamin
30. A dietician ina hospital is tounits of calcium, 10units of iron, 10 units of A, 20
gm. of food P contains 20 10 units of calcium, l0 units of
units of cholesterol. Each gm. of food Q contains cholesterol. Each gm. of R contains 10
iron, 20 units of vitamin A, and 24 units of vitamin A, and 18 units of cholesterol. If
units of calcium, 10units of iron, 10 units of
of calcium,200 units of iron, and 240
the minimum daily requirements are 300 units should be used to meet the minimum
food
units of vitamin A, how many gms. of each the cholesterol intake ? What is the
requirements and at the same time minimize
minimum cholesterol intake ? The finished
blending three different raw materials.
31. A product is manufactured byquality following data, what is
product should meet certain requirements. Given the which
quantity for raw materials to be blended
your recommendation with regard to minimum cost ?
willmeet the quality requirements with Minimum quality
Quality Contribution to quality by each unit of raw materials
C requirements
A B
characteristics
1 10
1
1 2 15
2 5
0
1
3
Cost of raw 5 3
2
materials per unit
(in Rs.)

for solution by L.P. model


32. Express the following inequations
Maximize Z = 2x1 - X2
subject to 3x1 + X2 =3
4x1+ 3x9 26
OPERATIONS RESEARCH
:PROBLEMS
3-62
X1 + 2x9
s4 &SOLUTIONS
20
X}>0, xy
linear programningproblem:
following
33. Conslder the +4Ox, +120x3 - 10x,
Maximize 2 80x constralnts
subject to the
5x4 S150
x + 2x2 +xg + 70
x) - Ax3 + &x4
2120
6x +7xy +2x3 -x4
X|, X2, X3, X4
20
equalities by adding the appropriate
constraints to
Convert variables. Also, add the new variables into the problem's objectiveor
(0 artificial these Surplus,
t
tableau for this problem. DO NOT SOL
function.
the complete initial simplex
(0 Set up difterent mines that produce a given kind
of a
two
34. A certain mining company owns different
havedlte.
parts of the country and hence classes : Hhel
The mines are located in three
production capacities. After crushing,There the ore is graded into of o
Medium grade and Low grade. is some demand for each qrade tons
grade,
company has contracted to provide a smelting plant with 12 It costo
The mining 24 tons of low grade ore per dayweek.
high grade, 8 tons of medium graderunand the first mine and Rs. 1000 per high to run the
the company Rs. 2000 per day to grade,
second. However, in a day's operation,low the first mine produces 6 tons of produces
grade ore while the second mine
2 tons of medium grade and 4 tons of grade and 12 tons of low grade ore.
daily 2 tons of high grade, 2 tons of medium
be operated in order to fulfil the
How many days a week should each mine
company's order most economically ?
amount of Rs. 30,000 in fixed income
35. A retired person wants to invest upto an in two bonds : bond Ayielding 7% and
securlties. His broker recommends investing decides to invest at most Ks.
bond B yielding 10%. After some consideration. he amout
also wants the bond
12,000 in bond B and at least Rs, 6.000 in the bond A. Heinvested in the B.
Invested in the bond Ato be at least equal to the amount
What should the broker recommend if the investor wants to maxim1ze nis reu
investment ?
information cn
36. Acandidate appearing for the competitive test finds the following
seeing the question paper.
) the total time is 3 hours,
(() the paper has two sections section A and section B questions
() to answer at least two questions from section A and at least three
from section B
15marks
(o) Aquestion of first section carries 10 marks and that of second section Bis25
(u) time to answer a question of section Ais 15 minutes and that of section
minutes
(u) The maximum number of questions to be
answered is 10. answerarnd
Find how many questions trom each section the candidate should
what are the maximum marks ? buses
20
37. Afactory manufactures two types of buses () Ordinary luxury
machinehous
buses and
machine hours are needed to manufacture one ordinary bus and 60 (i1) J
andthaton
are spent on one luxury bus, Cost on an lakhs ordnar
luxury bus is Rs. 60 lakhs. According to a ordinary bus is Rs. 10 at least3thelabur
and 2luxury buses have to be government regulation a law ofcapucito
board not more manutactured daily. Also due to
the factory is tothan
320 all not hours
produce inmachine are available
more than 10 newin one aday.
buses
the
day.Iffind houw
buses of each type should be
manufactured to spend minimum cost.
LINEAR PROGRAMMINGI 3.63

a special
mixture of
38 XYZ chemical
chemical chemical Bmust
Aandcorporation for aproduce exactly
customer. 1,000 A
Chemical kqs.costsof
Rs. 50 per kg, and
chemical B costs Rs. 60 per k. No more than 300 kgs. of chemical Acan be used
the
and at least 150 kg of chemicalB must be used. Using simplex method. determine
least cost blend of the two ingredients.
9. A company planning an advertising campaign to attract new customers wants to
place a total of at most ten advertisements in three news-papers. Each advertisement
in A costs Rs. 2000and willbe read by 20,000 people. Each advertisement in Bcosts
Rs. 2000 and will be read by 5000 people. Each advertisement in C costs Rs. 1000
to
and will be read by 15000 people. The company wants at least 1,60,000 people
read its advertisement. How many advertisement should it place in each paper n
order to minimize the advertising cost ? What is the minimum cost ?
A0. The Indian chemicals Pvt. Ltd. must produce 10.000 ka of a special mixture tor a
customer. The mix Consists of ingrediernts X,. Xo and Xa. X, costs Rs. 8 per kg, A2
costs Rs. 10per kg and X3 costs Rs., 11 per ka. No more than 3,000 kg of X1 can
be used and atleast 1500 kg of X must be used, Also. at least 2000 kg of X3 1s
required.
(a) Calculate the number of kg for each ingredient to use in order to minimize total
cost for 10,000 kg.
(b) Calculate the lowest total possible cost.
and Dwhich have
41. A transistor radio company manufactures four models A. B.A, BCand C respectively
profit contributions of Rs 8, Rs. 15 and Rs. 25 on models
time
and a loss of Re l on model D. Each tupe of ratio requires a certain amount of
for the manutacturing of components for assembling and for packing. Specially a
dozen units of model A require one hour of manufacturing, two hours for assemtbling
and one hour for packing. The corresponding figures for a dozen units of model B
of model
are 2, 1 and 2 and for a dozen units of C are 3. 5 and 1, while a dozen units
D require 1hour of packing only. During the forthcoming week, the company will be
able to make available 15 hours of manufacturing, 20 hours of assembling and 10
hours of packing time. Obtain the optimum production schedule for the company.
sold to the
42, Ashok Chemicals Co. manufactures two chemicals A and B which are demand,
manufacturers of soaps and detergents. On the basis of the next month's
and B should
the management has decided that the total production for chemicals A
be at leastAmust kilograms. Moreover, a major customer's order for 125 kgs. of
350 also
product be supplied. Product A requires 2 hours of processing time per
k. and product B requires one hour of processing time per kg. For the coming
month, 600 hours of processing time are available. The company wants to meet the
above requirements at a minimum total production cost. The production costs are
Rs. 2/- per kg. for product Aand Rs. 3/- per kg for product B.
Ashok Chemicals Co. wants to determine its optimum productwise and the total
minimum cost relevant thereto.
(0 Formulate the above as a linear programming problem.
() Solve the problem with the simplex method. (Delhi Uniu. M. Com., 1998)
(ti) Does the problem have multiple optimum solutions ?
43. Given the following initial simplex tableau :
S1 S A, Az
Baslc C 15 25 -M -M Quantity
uariables
-M 6 -1 0 1 20
A,
5 0 1 30
-M 3 -2 0 0 1 18
Az
-10M 4M M -M -M -38M

G -Z15 +10M 25 +4M -M 0 0 0


OPERATIONS RESEARCH:PROBLEMS &SOLUTIONS

3-64 tableau
oiginal problem represented by the above
(0 Write down the solution of this problen.
Is it i a unique solution ? Why ?
optimum
(i9 Find outthe shadow prices of the resources in the above mentioned problem?
(ii) What are the products A& B.
Product Ayields a contribution of
Rs.
manufactures the profit and fixed costs. It
44. Acompany product B Rs. 40 per unit towards not exceed 20, sales ofis estimated
60 per unit &
product Afor the coming month will product B
that sales of estimated but the company does have a contract to supply atleast 10
have not been customer. Machine hours available for the coming month are 100
units to a regular require 4 hrs. & 2 hrs. respectivelyto produce one unit. Labour
and product A& B A and B require 4 hours and 6 hours of ak
hours available are 180 & products are restricted to 40 units and the two products each
respectively. Materials available Determine the optimum product mix.
use one unit of material per unit. you the following information regardes
office furniture provides
45. A firm manufacturing availability and profit contribution
resource consumption and Usage per unit Daily
Resources availabilty
Tables Chairs Bookcases
4 640
Timber (cu. f) 2 540
Assembly department (man-hours) 4 6
1 100
Finishing department (man-hours) 1
30 20 12
Profîit contribution per unit (Rs.))
Minimum production requirement 50
The firm wants to determine its optimal product mix.
help of the above data.
(0 Formulate the linear programming problem with the productmx
i) Solve the problem with the Simplex Method and find the optimal
and the total maximum profit contribution.
(i) ldentify the shadow prices of the resources.
(iu) What other information can be obtained from the optimal solution of te
problem ? (Delhi Uniu. M. Com. 1990)

HINTS AND ANSWERS


L (a Ans. 3, 4: 160 (b) Ans. 6, 12; Rs. 120
2 (a Ans. 30, 70:540 (b) Ans. 50,800: Rs. 1850 rate of Rs.
3 Hint. Since any amount of unused time on machine M, can be rented out at a thetotal
1-50 per hour, the total rent obtainable wouldbe: 1-5 [1,000-(x1 + x)]. Thus
profit would be equal to 10x, +12x, +1500- 1-5x, - 1-5x,. The LP model 5·
Maxirize Z= 8-5x1 +10 5x) +1500
subject to the constraints
2x +3x, s 1500, 3x +2%,s 1500, x, +x, s 1000: x. X,2 0 l and B Total
Ans. Optimum solution is to produce 300 units of
each of the productsA
profit = 8-5x300 + 105 x 300 + 1500 = Rs. 7,200.
5 Ans. 120, 150; 1320 valves
& Hint. Let x,, x, and 6 Ans. 15, 28: Rs.. 1065 and
x, be the required rings
quantities of pistons,
manufactured and sold. Then the appropriate formulation of LP model will be
Maximize Z= 10x, +6x, +4x,
subject to the constraints
x+ x+ xS 100, 10x +4x, +5x, S600; 2x, +2x, +6x, S300; X , ,20 cqualities
Introducing
the [lack
above problem variables in the above convert them into
in standard form is : constraints to
Maximize Z= 10x +6xt 4x, +0.s, +0.s, +0.s,
subject to the
constraints
x+x,ty t S, +0.$,+0.s,= 100, 10x, +4x, +5x,+ 0.s, +s+0.s, =600:
2x +2x, +6x, +0.s, +
0.s,+ S=300, x X2 y S,, s,, s,20
UNEAR PROGRAMMING-|I
3.65
The initial feasible solution of the
solution gives s, = 100, s, = and formulated nroblem is: x= s =
s, = 300, This solution for further 2 =0. The miLa
be presented in the following600
tabular form. improvement can
SIMPLEX TABLEAU I, II &III
Basic 10 6 Mint
C Solu 4
mum
variables tion
ratio
S S S
S 100
1 0 100
S 600 10 4 5 60’
S 300 2 2 6 1 150
0 0
10T 6 4
S 40
6/10 5/10 -1/10 400/6’
10 60 1 4/10 5/10 1/10 150
180 12/10 5 300
-2/10
600 10 4 5 1
21 -] -1
6 400/6 5/6 10/6 -1/6
10 200/6 1 1/6 4/6 1/6
S 100 0 4 -2
4400/6 10 6 40/6 20/6 4/6
G-Z 0 -16/6 -20/6 -4/6
Since all entries in Crz row are either zero or negative, the final table gives the
optimum solution of theformulated problem and is given by : x,= 100/3, X, = 200/3 and
=0.
9. Hint. (a) Let x, X, and x denote the number of boats of ROW BOAT, CANOE and KAYAK
respectively made every month. Then, the formulation of the required L.P. model will
be as follows :
Maximize Z= 4000x, +2000x, + 5000x,
subject to the constraints
12x + 7, + 9x, s 1260 (Labour hours constraint)
22x + 18x, + 16x, S 19008 (Wood board feet constraint)
2x, + 4, + 3%s 396 (Screws (in kg) constraint)

(b) Introducing slack varlables s,, S, and s, and presenting the problem in the standard
form, the above formulated problem becomes :
Minimize Z= 4000x, +2000x, +5000x +0.s, +0.s, +0.s,
subject to the constraints
12x + 7y+ 9x,+ S, + 0.s,+ 0.s, = 1260
22x, + 18x, + 16x, + 0.s, + S,+ 0.S, = 19008
2x + 4x, + 3x,+ 0.s, + 0.s,+ Os,= 396

The initial solution and subsequent simplex tables are given below :
SIMPLEX TABLEAU I, II, & II
G’ Basic Solu 2000 5000
Mini
4000 0
Varla tion mum
bles values S S ratio

1260 12 7 0 0 140
S
19008 22 18 16 1 0 l188
396 2 4 3
S 132
OPERATIONS RESEARCH::PROBLEMSS&SOLUTONS
3-66
0

4000 2000 ‘s000 0


C,-Z 1 -3
72 6 5 124
34/3 -10/3 0 -16/3 1491
16896
0
4/3 1 0 0 1/3
5000
132 2/3 198
5000 0 5000/3
660000 10000/3 20000/3
12000/3 -14000/3 0 -5000/3|
C-2 0 1/6 -3/6
12 -5/6
4000 x
16760 55/9 -34/18 1 1/3
S
124 17/9 1 2/18 2/3
5000
668000 4000 55000/9 5000 1000/9 0 4000/3
-49000/9 -1000/9 0 -4000/3
G-4
the final table will gtve the
Since all entries in the C,-Z, are either zero or negative,124.
optimum solution which is given by: x, = 12, x, = 0 and x, =
boats only. The resulting
(0 The company should produce 12 Row boats and 124 Kayak
revenue will be : Rs. 4,000x 12 + Rs. 2,000 x0+ Rs. 5,000 x 124 = Rs. 6,68,000.
utilized. It is not used to the extent
(i) Wood (Board feet) is the resource that is not fully board feet.
of 16,760board feet, Le., spare capacity of resource wood is 16,760
16 x 124).
(ü) Total wood used for the production of optimum mix is : 22 x 12 + 18 x0+
10. Hint. Maximize Z= 5x, +4x, +6x,
subject to the constraints
X+x,+x, S 100;3% +2x, +4x, s210; 3x +2%, S 150;x, %, , 20
11. Hint. Let x,, x, and x, denote the number of units for the productsA, Band Crespecuvey
The appropriate mathematical formulation of the given problem is :
Maximize Z= 2100x, + 1600x, + 1300x,
subject to the constraints
60x, +50x,+40x, S4000; 3x + 2x%, + x%s144; x, X,, X520
Introducing slack variables s, 2 0, s,2 0, the iterative simplex tables are :
SIMPLEX TABLEAUI. II &II Minimum
Basic Solution 2100 1600 1300 0 ratio
S; S
variables values
4000/60
4000 |8( 50 40 1
144/3
144 3 2 1
X, enters 0

& s, leaves G-Z 2100‘ 1600 1300 l100/20


1100 8 10 (20+ -20 48
2100 X 48 I/3
X, enters
- I/3 1/3
100800 2100 1400 700 0 700
& s, leaves C-Z 0 200 -700
600t
1300 56 0 I/2 1 1/20 -1
2100 X 88/3 2/3
1/2 -l/60
134400 -100 1700 100
1300 30
G-Z 8 100 0 -30 -100
LINEAR PROGRAMMING--| 367

The optimum solution is to produce 88/3 units of product A, 56 units of prodtG


and should not produce Bin order to achieve a maximum proft of Rs. 1,34,400. Pure
since opportunity cost of s, is 100,department Ilshould be glvern
12. Hint. Maximize (total profit) Z prlority.
=30x, + 40x,
subject to the constraints
60x + 120x, s12,000; 8x +5x, s600; 3x +4%, s 500: x,. X,20
Introducing
form becomesslack
: variables s, >0, s, 20, the linear programming problem in standard
Maximize 2= 30x, +40x, +0s, +0s, +0s
subject to the constraints
60x +120x, +s, =12,000; &x, +5x +S, =600; 3x, +4x, + S, =500, x,, Sy Sy
An obvious initial basic feasible solution is obtained by setting x, = X 0 Soe
s,= 12,000; s, =600 and s, =500.
INITIAL SIMPLEX TABLEAU I(Non-optimum solutlon)
30 40
Basic Minimum
Solution S S, S
ratlo
variables values
12,000 60 120® 1 0 1004- key row
600 5 0 1 0 120
Sn
500 3 4 0 1 125
0
30 40
G-Z
FINAL SIMPLEX TABLEAU II (Optimum solution)
C 30 40

Basic Solution X S S3
variables values

1000 2
1
40 11 165

200 1
30 11 132

900 17
S 11 660 11

46000 34 20
30 40 0
11 132 11

34 20
G-4 132 11

Since all entries in the C-Z row of the tableau IIl are either zero or negative,an
optimum solution has been obtaiñed which is given by :
1000 200 46000
and x,= with maximum Z=
13. Ans. (0Maximize Z= 10x, +25x, +3x
subject to the constraints
3
3x, +10x%, +x,s 30,000,5x, +Bx +xS40,000: 0%4+ 0%s 120; x,.4 y20
Z= 12,000 (úo No.
(t) x, = l200, x, = X, =0with maximum
OPERATIONS RESEARCH:: PROBLEMS &
3-68

14. Hint. Maximize Z= 25x, + 40x,


SOLUTIONS
subject to the constraints
0-50x, + 025x,s 2,200
4x + 7x%,s 28,000
0-20x, +0-20x, s 1,400
0
Introducing slack variables. the above LP problem in standard form becomes :
Maximize Z= 25x, +40x, +0:s, +0S +0$,
subject tothe constraints
0-50x, +0-25x, + s, = 2,200
4x +7x, +S,= 28,000
0-20x +0-20x, + S,= 1,400

The initial feasible solution is obtained by setting x, =X, = 0.


SIMPLEX TABLEAU I, II & III
Basic Solution 25 40
Minimum
S, S, S3 ratio
variables values
S 2,200 0-50 0-25 8.800
28,000 4 7* 4,000
1,400 0.2 02 1 7,000
X, enters
s, leaves G-Z 25 40

40 4,000 4 7,000

S 1,200 5 1 3.360+
14 28
600 3 1
S 35
7,000
35
X, enters 1,60,000 160 40
7 40
7
& s,leaves G-Z 15 40
7
25 X 3,360 14 1
1 0
40 10
2,080 8
1
5 5
312 6
25 50
1,67,200 25 40 6 0
2
G-Z 0 0 -6
2
Since there is no positive isoblained
number
and is given by : x, =3,360 ; x, = in the C, - Z, row, the solution
(io Used and unused optimum
2,080 with max.Z= Rs. 1,67,200.
resources
Resorces
Leather
Used Unused
Total
0-50x 3.360 +0-25 x2,080 = 22,200kg
Wood 2,200 kg, 0 kg.
Ghue 28,000 sq. mt. 28,000sq. ml.
0 sq. mt. 1.400litres
1,080 litres 312 litres
(i) Shadow prices are Rs. 6, Rs. 2 and Rs, 0 respectlvely.
16 Ans. In order to achieve a maximum profitforof leather.
Rs, 1,500,
Woodtheand
school
Glue should make 20
proll
chairs of type A. 20 chalrs of type B maxtmum
of Rs. I,500. and no chalr of type Cto
achieve a
UNEAR PROGRAMMINGI 3-69

16 Hint. SIMPLEX TABLEAUI. II & III


500 600 1200 Mini um
Basic Solution ratio
S, S
Uariables values
160 2 4 6 160/6
3 4 1 120/4
120

500 600 1200


C 80
1200 160/6 1/3 2/3 1/6 0
1 8
80/6 5/3* 4/6 -2/3
S
32.000 400 800 1200 200
100 ‘ -200
C-Z - 200
27/90 -1/5
24 72/40
8 -2/5 -2/5 3/5
X
32.800 760 12000 160 60
500 -160 -160 -60

The optimum solution is : x=8,x=24 and maximum Z=32.800.


The company should divide the supply of previous metals as follows :
Product Units to be Supply of Platinum Supply of Gold
manufactured (gm) (gn.)
8x2= 16 8x3= 24
A
Ox4= Ox2=
B Nil
24x6= 144 24x4= 96
C 24

160 120
Total
and optimum profit = Rs. 32,800.
20. Ans. Maximize Z'=Z+ 150= 4x, +3x, + 5x
subject to the constraints
2x, +3xG +2x,s 400
3x, + 2x+2%s 350
x, +4, +2%s 300
(787-50 + 150)
=0 andxa= 137-5 with max. Z= Rs.
The optimum solution is : x, = 25, x, and x, the number of
number of acres of crop A to be cultivated formulation of the LP
21. Hint. Let x, denotes the appropriate mathematical
acres of crop Bto be cultivated. The
problem is:
Maximíze Z= 150x, + 200x,
subíect to the constraints
X,+xs 150
40x, + 60x, s 7400
20x, + 25x, < 3300
70

in the
problem, we rewrite the fourth inequality
In order to solve this nonstandard
-x, S-70
form :
becomes :
Introducing slack variables, the problem O-s,
Maximize Z= 150x, + 200x, + 0-s, +0:s, + O-s, +
150
subject to
40x, + 60x, + S,= 7400
20x + 25x, + s,= 3300
+ S,= -70
OPERATIONS RESEARCH: PROBLEMS &
3.70
$OLUTIONS
SIMPLEX TABLEAU-I, (Non-optimum solution)
150 200
Solution
Basic
variables values
1
S S, S S
Minratlo
imum
150 150
S
S
7400 -88
40
20 25 1
185
0 S 3300 165
-70 -1" 70
S.
-150‘ -200
C-Z
SIMPLEX TABLEAU I(Non-optimum solution)
150 200
Basic Solution
S S S
variables values

4 0 5
S
12
40 1
Sy 5
4
200 76 0 25 5

150 70
0 8 10
G-4
This last table is in final from and the optimum solution is:
x=70, x =76, S, = 4, S, =40, S,=0, S,=0 and Z= 25,700.
Thus, by cultivating 70 areas of crop A and 76 acres of crop B, the farmer will attain a
maximum profit of Rs. 25,700.
22. Hint. Maximize Z= 3x + 4x +6x,
subject to the constraints
80
4x +x +6x, s 25x 16x3 x 100 or 960
80
5x +3x, +% S 25x 16x2x 100 or 640

80
x+21,+3 s 25x 16xl x 100 or 320

By introducing slack variables standard form.


S,. S, S, Write the above constraints in
FINAL SIMPLEX TABLEAU - II (Optimum solution)
4 o6 0
G Basic Solution
Cp variables values S
13
640
7 4 0 1
1
800
3 X 1 14
7 14
6 480 1
7 -14
5280 3 14
7 6 14 21
3 14
C-Z 1
2
0 14
800
Ans. x, = 7y=0, x=0. x,= 480 5280
7 with maximum Z=
7
23. Ans. 4 =430, 4,= 460, a, =420; b, =-
24. Hint. (0 Yes, the given b=b =2: c =-1, c, =-2, 32 lastroward
solution
less than or equal to zero. is optimum, since all the elements in
the
are
(in No, because the andS,
strictly negative. elements in the last row under non-basic variables X
INEAR PROGRAMMINGI 3.71

tA No, because none of the basic variables x, or s, has zero value.


(tu) Ves. because the values of basic variables x, and s, are non-negative. of slack variable s, is
(v) Machine A is being used to the full capacity because the value tíme (in hours/week)
ero in the optimum simplex of table. This indicates that the entire
activities the model.
is consumed by the
(v) From the above table,
the element in the last row under x, is -1, therefore. an
increase of x, from its current zero level to a positive level will
cause the reduction in
Hence in order to ensure that there
the total profit at the rate of one rupee per week. of x, should be increased by at least one
should not be any reduction in profit, price
rupee.
of x, units and 5 units of x, should be
(vi) From the given solution, we find that none
produced. The total profit is 4 x0+5x 10 = Rs, 50.
price of hours on machine A is Rs. 5 per
(vii) From the last row, we find that shadow
Rs. 2x5= Rs. 10.
hour. Hence, reduction in profit of 2 hour shut down=
on machine A and B being Rs. 5 andforRs.another O respectively,
(ix) The shadow prices of hours prices one would be prepared to pay hour of
these will be the maximum
capacity for these two machines.
,= 8 X= 56 with maximum Z=
25. Hint. (i) x=0,
of basic variables x,, X and s, are non-negative.
(i) Yes, because the values x,, S, and s, are
(ii) No, because theelementsin the last row under non-basic variables
strictly negative.
Department I.
(iv) 1/6, 11/3, and0. () is 19/3 : therefore, an
From the above table, the element in the last row under x, mean the reduction in
(v) zero level to a positive level will
increase of x, from its current there should not be
total profit at the rate of Rs. 6-33. Hence in order to ensure that
the should be increased by Rs. 6-33.
any reduction in profit, price of x
mathematical formulation of the problem is :
27. Hint. The appropriate
Minimize Z= 3x, + 4x
subject to the constraints
2x, + 3x, 90
4x +3%2 120

respectively.
x, denotes the number of units of liquid and dry fertilizer
where x, and problem is as
variables, the standard form of LP
Introducing surplus and artificial
follows :
+ M.Az
Minimize Z= 3x, + 4x, + 0-s, + O-S, + M.A,
subject to the constraints
2x + 3 - S, + A, = 90
4x + 3x,-S,+A,= 120
given below :
inftial simplex tableau giving the initial solution to the problem is
The INITIAL SIMPLEX TABLEAUI
M M Minimum
3 4
Basic Solution ratio
S S A A,
variables values
-1 45
90 2 3
M A, -1
M A, 120 4"

6M 6M
3

- M - M M M
30|
3-6M 4-5M M M Key row
G-3 " Key number
Key column
3-72 OPERATIONS RESEARCH: PROBLEMS&SOLUTIONS
SIMPLEX TABLEAU II (x, enters and A, depors
Basic Solution
M M
C,J variables values
A, Ag Minratloimum
M +A, 30 -1
2 1 20 (Key row)
3
3 30 40
3M 9 M3
-M M
24 2 4
7 3M 3 M
G-Z 4 2
M
4 2
*Key number ‘
Key column
FINAL SIMPLEX TABLEAU III (, enters and A, departs)
Basic Solution 3 4 M M
S S A, A, Minimum
CGt variables values ratlo
4 20 2
3
3 X 15

125 7
4 3
6 6
G-Z 7
6 6
Since all entries in the C, - Z, row are either zero or positive, the optimum solution nas
been arrived at and is given by : x, = 15, x, 20 with minimum Z= Rs. 125.
28. Ans. 10kg. each of Type I and Type II.
30. Hint. Let x,, xy, and x, be the number of gms. of food P., Q and
Rrespectively.
Minimize Z= 20x, + 24x, + 18x,
subject to the constraints
20x +10x, +10x, 300
10x + 10x, + 10,2 200
10x, + 20x, + 10x, 2 240

FINAL SIMPLEX TABLEAU- V (Optimum solution)


M
G’ Basic Solution 20 24 18 M M
A.
C, variables values S S, S A, A,
20 x 10
1 1
-
18 10 10
6
3
5 10 10
24 4
10 10 i0
404
20 24 18 4 3
5 -1
5
G-4 0
4
5
Ans. 10 gms. of P 4 gms. ol 9, 6
gms. of R ;
31. Hint. Let x,. X, and x, representthe quantitywith
(no.minimum
of units) cost
of raw materials
be blended. Thern the
of 404 unils. A, Band Cto
Minimize Z= 2x,mathematical
+5x, +3x, formulation of the problem is :
subject to 3x, +Ox, tx, 210; 5x,+ x, +
Introducing slack and artilicial variables, the 2xz15, x, +2x, +Ox, 2 8: X i n.standard
form as , problemcan be represented
Minimize Z =2x, +bx,t 3x, t Os, +Os, +0s, +
MA, + MA, + MA,
INEAR PROGRAMMING-| 3-73

subject to the constraints


3x, + Ox, + X,-s, + A, = l0
5x, +x, +2x,- s, + A, 15
x, + 2x, + Ox,- & +A,= 8

Optimum solutlon is : x8, x, X, Oand minlmum 2- 16.


34. Hint. Minimize Z - 2000x, + 1000x, 35. Hint. Maximtze Z= 0-07x, + 0- 10x2
subject to the constralnts subject to the constralnts
6x, + 2, 12 X, + x s 30,000
2x, + 2x 8 S 12,00O0
4x, + 12x, 24 Xy 6,000

Ans. x,= 12,000 and X,= 8,000


36. HInt. Maxmize Z = 10x, + 15x,
subject to the constraints
x22, x,23, 15x,+25x,S 180, x +xs 10:x,, x20
Ans. x, =7, =3 with max. Z= 115.
37. Ans. 3 ordinary type buses and 2
luxury type buses at minimum cost of Rs. 150 lakhs.
38. Hint. Minimize cost, Z= 50x, + 60x,
subject to the constraints
X + x,= 1000, x s300, x, 2150; x 0, , 20
where X, = Number of kgs. of chemical A and
X, = Number of kgs. of chemical B.
constralnts can be
Introducing slack, surplus and artiflcial variables, the above
rewritten as follows :
Minimize cost, Z= 50x, + 60x,+ 0s, + 0s, + MA, + MA,
follows :
The complete set of constraints can now be expressed as
x, + + Os, + Os, + A, + OA, = 1000
lx, + Ox,+ 1s, + 0s, + 0s, + OA, + 0A,= 300
Ox + x, + Os, - S, + 0A, +A, = 150
xj, Xg S, S,, A,, A, 2 0
INITIAL SIMPLEX TABLEAU- I (Non-optimum Solution)
50 60 M
Baslc Solution Minium
G’ values S S S4 ratio
Varlables

1000
1,000
M A,
S, 300

M 500 1504
-1

2M -M M M
M
-M +50 -2M+ 60 0 M

Solution)
FINAL SIMPLEX TABLEAU-IV (Optimum
50 60 M
Bastc Solution
G’ values X A, A,
varlables
-1
550
0
50 300
700 - 1
60
5700 50 60

G-4
infeasible. solutlonis table,
the llnal present
in variable
is artúficlal Since
M- 0 9 9 3 6
0 20 M 40 M G-Z
0
M 0 99 63
0
M 20 M 40 M 40 60
-1 0 0 0 0 3
10 A, M-
0 0 1 3 0 1 3
10 60
0 0 9
0 0 3
0 1 20
00 9 1 0 3
2
40
0 1 0 3 0 0 3
10
S S values variables
-M-M 0 0 0 40 Solutton
60 Basic
Solution) (OPTIMUM TABLEAU-IV SIMPLEX FINAL
A, +
40 = XtxytS,
10 = A, +S,X1-
20 = S +
180 = S +6x, + 4x
100 = S, + 2x + 4x
constraints the subject
to
MA, MA,- 0.-s, 0.+s, 0.+s, +0.s, +0.s,+ 40x, 60x,+ Maximize
Z=
varlables,
get we artificial andsurplus slack,
Introducing
20x,x,and 10 %,2 S20, x%
40, =x, +180,x6x,s +4x
100, S2x, 4x,+
constraints the subject
to
Z=
40x, 60x,+ Maximize 4.
120 maximum
Z= and ,=,4=.
=0 X =x,Ans.
5
20 Xgy 10;X, +x= xy+2x, +x,
20, 5x,= +x,+2x15, =3y +2x, + x
constraints the subject
to
25xg-X 15x,+ +8xZ=
Maximize
respectvely. produced betoCandD B,A,model Unitsof =X Xy. X,. X,. Hint. 41.
96,000. Rs. (0
2,000 x,= and 5,000 3,=0x,
00; =x,(0 40.
12,000. Rs. minimum
Z= with 8 =x,=0, x, 2, =Ans.x
1,60,000 x,
215,000 +5,000x,
+ 20,000x,
10 S 4
constraints the subject
to
1000x, 2000x,+ 2000x,+ Z=Minimize
respectively.
annnewspapers
d BA, advertisements
in numberof the bex,and X,x,Let Hint.
39
A :PROBLEMS& RESEARCH OPERATIONS 3-74

You might also like